Intro Nursing Fundamentals 3

¡Supera tus tareas y exámenes ahora con Quizwiz!

1. The student learns that which is the best definition of a public health nurse? a. Works with the public. b. Works in public areas. c. Works with the greater community. d. Works with public funding.

ANS: C A public health nurse works with communities as a larger whole and is concerned with specific target or vulnerable groups within that community. The other options are inaccurate.

Upon assessment a nurse discovers postural abnormalities on several patients. Match the abnormalities to the findings the nurse observed. a. Lateral-S- or C-shaped spinal column with vertebral rotation b. Exaggeration of anterior convex curve of lumbar spine c. Increased convexity in curvature of thoracic spine 1. Lordosis 2. Kyphosis 3. Scoliosis

1. ANS: B 2. ANS: C 3. ANS: A

13. The nurse is preparing to reposition the patient in bed. What is the first step in this process? a. Position the patient's arms across his/her chest. b. Lower the side rails. c. Grasp the draw sheet. d. Raise the bed to a working height.

13. The nurse is preparing to reposition the patient in bed. What is the first step in this process? a. Position the patient's arms across his/her chest. b. Lower the side rails. c. Grasp the draw sheet. d. Raise the bed to a working height. ANS: D Raising the bed to a working height is the first step before beginning the procedure. Proper positioning of equipment prevents provider discomfort and reduces the chance of possible injury. Then lower the side rails as appropriate and safe and ensure that the bed wheels are locked. Then you can have the patient position his/her arms and/or grasp the draw sheet.

14. A nurse is working with a student nurse. 2 patients are in Airborne Precaution and Contact Precaution. What are the appropriate teachings?

14. A nurse is working with a student nurse. 2 patients are in Airborne Precaution and Contact Precaution. What are the appropriate teachings? Have patient with airborne precaution wear mask Use nursing knowledge to prevent the spread of disease

15. Patient will be pain free in 48 hours (terminal disease, chronic pain). Nurse gives medication as ordered.

15. Patient will be pain free in 48 hours (terminal disease, chronic pain). Nurse gives medication as ordered. Unrealistic goal

8. A hospitalized patient is fall risk. What should the nurse ask?

8. A hospitalized patient is fall risk. What should the nurse ask? Are you more unsteady when standing after getting out of bed?

9. A patient has an acute change in neurologic. What should the nurse do?

9. A patient has an acute change in neurologic. What should the nurse do? Perform a neurologic assessment

13. Nontherapeutic communication may lterfere with professional nursing care by hindering the patent-nurse relationship. Which of the following are examples of nontherapeutic communication? (Select all that apply.) A. A nurse provides education on smoking cessation: "The same thing happened to me and I was able to quit" B. A nurse attempts to distract a patent at the end of their life: "Let's focus on your walking for the day, not your worries about death. C. A nurse states during morning report, "The patent should not get an abortion because It is wrong." D. A nurse is discussing care options with the family of a patient: "What have your experiences been like with home healthcare in the past?

A. A nurse provides education on smoking cessation: "The same thing happened to me and I was able to quit" B. A nurse attempts to distract a patent at the end of their life: "Let's focus on your walking for the day, not your worries about death. C. A nurse states during morning report, "The patent should not get an abortion because It is wrong."

7. After formulating several diagnoses, the nurse does not understand the reason for some of the discrepancies in the client's lab values and diagnostic tests, when comparing to norms and standards. Which action should the nurse take? A. Consult other professionals and colleagues. B. Verify the information with the client. C. Improve critical thinking skills so answers come more easily. D. Compare all findings to the national norms and standards.

A. Consult other professionals and colleagues.

19. An error occurs as the result of the lack of a double-check process on dosing of a high-risk opiate patient becomes oversedated, necessitating reversal of the opioids in order to regain a regular the following should the nurse include in the incident report? (Select all that apply.) A. Describe factors that led up to the incident. B. Document patient assessment findings following the error. C. Detail contributing factors in the patient chart. D. Include Intervention needed to reverse oversedation in an incident report.

A. Describe factors that led up to the incident. B. Document patient assessment findings following the error. D. Include Intervention needed to reverse oversedation in an incident report.

22. A nurse gives an incorrect medication to a patent without doing all of the mandatory checks, but the patient has no ill effects from the medication. What actions should the nurse take after reassessing the patient? (Select all that apply) A. Notify the health care provider of the situation. B. Document in the patient's medical record why the medication omission occurred. C. Document in the patient's medical record that an Incident report was filed. D. Continue to monitor the patient for any untoward effects from the medication.

A. Notify the health care provider of the situation. D. Continue to monitor the patient for any untoward effects from the medication.

30. The nurse is caring for an older-adult patient admitted with nausea, vomiting, and diarrhea due to food poisoning. The nurse completes the health history. Which priority concern will require collaboration with social services to address the patient's health care needs? A. The electricity was turned off 2 days ago. B. The water comes from the county water supply. C. A son and family recently moved into the home. D. This home is not furnished with a microwave oven.

A. The electricity was turned off 2 days ago.

35. Models or health offer a perspective by which to understand the relationships between the concepts of health, wetness, and illness. Why are nurses in a unique position to assist patients in achieving and maintaining optimal levels of health? Select all that apply.) A. Understand the challenges of today's health care system. B. Identify actual and potential risk factors. C. Have coined the tam "Illness behavior" D. Can minimize the effects of illness and assist to the return of optimal health

A. Understand the challenges of today's health care system. B. Identify actual and potential risk factors. D. Can minimize the effects of illness and assist to the return of optimal health

17. A day one postoperative patent has the following abnormal assessment findings: pain 9/10, diminished lung sounds, hypoactive bowel sounds, and a saturated abdominal dressing and bed sheets. Which assessment would be the nursing priority? A. Wound care B. Bowel sounds C. Lung sounds D. Pain

A. Wound care

3. A nurse is planning primary prevention activities. Which activity would the nurse include in this plan? a. Safer sex education for teens b. Mammogram screening c. Medication compliance d. Annual physical exams

ANS: A Primary prevention includes activities designed to prevent a disease or condition from occurring in the first place. Examples of primary prevention activities include vaccinations, wellness programs, good nutrition for health, and safer sex programs. Mammograms and physical exams are secondary prevention measures. Medication compliance would be tertiary prevention.

3. The nurse is preparing a smoking cessation class for family members of patients with lung cancer. The nurse believes that the class will convert many smokers to nonsmokers once they realize the benefits of not smoking. Which health care model is the nurse following? a. Health belief model b. Holistic health model c. Health promotion model d. Maslow's hierarchy of needs

ANS: A The health belief model addresses the relationship between a person's beliefs and behaviors. The holistic health model recognizes the natural healing abilities of the body and incorporates complementary and alternative interventions such as music therapy. The health promotion model focuses on the following three areas: (1) individual characteristics and experiences, (2) behavior-specific knowledge and affect, and (3) behavioral outcomes, in which the patient commits to or changes a behavior. Maslow's' hierarchy of needs is based on the theory that all people share basic human needs, and the extent to which basic needs are met is a major factor in determining a person's level of health.

38. A nurse is supervising the logrolling of a patient. To which patient is the nurse most likely providing care? a. A patient with neck surgery b. A patient with hypostatic pneumonia c. A patient with a total knee replacement d. A patient with a stage IV pressure ulcer

ANS: A A nurse supervises and aids personnel when there is a health care provider's order to logroll a patient. Patients who have suffered from spinal cord injury or are recovering from neck, back, or spinal surgery often need to keep the spinal column in straight alignment to prevent further injury. Hypostatic pneumonia, total knee replacement, and stage IV ulcers do not have to be logrolled.

30. A nurse is providing care to a group of patients. Which patient will the nurse see first? a. A patient with a hip replacement on prolonged bed rest reporting chest pain and dyspnea b. A bedridden patient who has a reddened area on the buttocks who needs to be turned c. A patient on bed rest who has renal calculi and needs to go to the bathroom d. A patient after knee surgery who needs range of motion exercises

ANS: A A patient on prolonged bed rest will be prone to deep vein thrombosis, which can lead to an embolus. An embolus can travel through the circulatory system to the lungs and impair circulation and oxygenation, resulting in tachycardia and shortness of breath. Venous emboli that travel to the lungs are sometimes life threatening. While the patient with a reddened area needs to be turned, a patient with renal calculi needing the restroom, and a patient needing range of motion, these are not as life threatening as the chest pain and dyspnea.

19. Which activity will increase the need for the nurse to monitor for equipment-related accidents? a. Using a patient-controlled analgesic pump b. Making an entry in a computer-based documentation record c. Using a plastic measuring device to accurately measure urine d. Removing medications from a manual medication-dispensing device

ANS: A Accidents that are equipment related result from the malfunction, disrepair, or misuse of equipment or from an electrical hazard. To avoid rapid infusion of IV fluids, all general-use and patient-controlled analgesic pumps need to have free-flow protection devices. Measuring devices used by the nurse to measure urine, computer documentation, and manual dispensing devices can break or malfunction but are not used directly on a patient and are considered procedure-related accidents.

13. A patient is diagnosed with pneumonia after an abrupt onset of fever, cough, and malaise. The patient is started on antibiotic therapy and is expected to improve in 2 to 3 weeks. Which statement by the nurse correctly identifies this illness? a. Acute b. Chronic c. Remission d. Exacerbation

ANS: A Acute illness is typically characterized by an abrupt onset and short duration (<6 months). Clinical manifestations of acute illness appear quickly. They may be severe or lethal, or they may soon resolve because they respond to treatment or are self-limiting. Chronic illness is characterized by a loss or abnormality of body function that lasts longer than 6 months and requires ongoing long-term care. Chronic health conditions may be controlled with lifestyle management or drug therapy, but they are considered to be irreversible. Chronic illness may be characterized by periods of wellness (i.e., remission) and exacerbation (worsening) of clinical manifestations, which can be life threatening. Individuals learn to adjust their lifestyles accordingly.

10. A home health nurse is assembling a puzzle with an older-adult patient and notices that the patient is having difficulty connecting two puzzle pieces. Which aspect of sensory deprivation will the nurse document as being most affected? a. Perceptual b. Cognitive c. Affective d. Social

ANS: A Alterations in spatial orientation and in visual/motor coordination are signs of perceptual dysfunction. Cognitive function is the ability to think and the capacity to learn; the patient is not disoriented or unable to learn. Affective problems include boredom and restlessness; the patient is participating in an activity. The patient is interacting with the home health nurse, so socialization is not a problem.

17. The public health nurse volunteers for a missionary group caring for Ebola patients in Africa. The nurse is reviewing the data using analytic epidemiology methods. What information does the nurse collect as the priority? a. Cultural norms in burial practices b. Genetic variables in disease acquisition c. Statistics related to incidence and prevalence d. Autopsy data on direct cause of death

ANS: A Analytic epidemiology hypothesizes why a disease is occurring in a community and looks at cultural practices, nutrition, and extrinsic factors such as the environment for links. Genetic variables and direct cause of death data are more related to epidemiology.

7. Aspects of safety culture that contribute to a culture of safety in a healthcare organization include which component? a. Communication b. Fear of punishment c. Malpractice implications d. Team nursing

ANS: A Aspects that contribute to a culture of safety include leadership, teamwork, an evidence base, communication, learning, a just culture, and patient-centered care. Fear of professional or personal punishment and concern about malpractice implications are considered barriers to a culture of safety. No model of nursing care has been related to a culture of safety.

44. The patient has been diagnosed with a spinal cord injury and needs to be repositioned using the logrolling technique. Which technique will the nurse use for logrolling? a. Use at least three people. b. Have the patient reach for the opposite side rail when turning. c. Move the top part of the patient's ÿÿÿêµÿj+__torso and then the bottom part. d. Do not use pillows after turning.

ANS: A At least three to four people are needed to perform this skill safely. Have the patient cross the arms on the chest to prevent injury to the arms. Move the patient as one unit in a smooth, continuous motion on the count of three. Gently lean the patient as a unit back toward pillows for support.

7. A patient who recently had a stroke is going to be discharged at the end of the week. The nurse notices that the patient is having difficulty with communication and becomes tearful at times. Which intervention will the nurse include in the patient's plan of care? a. Teach the patient about special assistive devices. b. Make the patient talk as much as possible. c. Obtain an order for antidepressant medications. d. Place a consult for a home health nurse.

ANS: A Because a stroke often causes partial or complete paralysis of one side of a patient's body, the patient needs special assistive devices. The nurse should include interventions that help the patient adapt to this deficit while maintaining independence. Teaching the patient to use assistive devices allows the patient to care for him- or herself. Making the patient talk can be inappropriate and demeaning. A home health nurse is not necessary as long as the patient is able to care for him- or herself. Instead of placing the patient on antidepressants, assist the patient in attempting to adapt behavior to the sensory deficit.

18. A patient diagnosed with chronic emphysema (lung disease) states ―I would be better off dead.‖ The nurse learns that the patient, has recently become unemployed because of oxygen dependency. The patient's spouse will have to go to work to support the family. Which action should the nurse take? a. Develop a plan of care for the family. b. Contact psychiatric services for a referral. c. Assure the patient that things will work out. d. Focus the plan of care solely on maximizing patient function.

ANS: A Because of the effects of chronic illness, family dynamics often change. The nurse must view the whole family as a patient under stress, planning care to help the family regain its maximal level of functioning and well-being. Psychiatric services may be a part of that plan but do not represent the entire plan. Offering false assurance is never acceptable. Focusing only on the patient will not help the family adjust.

39. The nurse is providing teaching to an immobilized patient with impaired skin integrity about diet. Which diet will the nurse recommend? a. High protein, high calorie b. High carbohydrate, low fat c. High vitamin A, high vitamin E d. Fluid restricted, bland

ANS: A Because the body needs protein to repair injured tissue and rebuild depleted protein stores, give the immobilized patient a high-protein, high-calorie diet. A high-carbohydrate, low-fat diet is not beneficial for an immobilized patient. Vitamins B and C are needed rather than A and E. Fluid restriction can be detrimental to the immobilized patient; this can lead to dehydration. A bland diet is not necessary for immobilized patients.

23. A nurse is caring for an older-adult patient on bed rest with potential sensory deprivation. Which action will the nurse take? a. Offer the patient a back rub. b. Hang a ―Do not disturb‖ sign on patient's door. c. Ask the patient ―Would you like a newspaper to read?‖ d. Place the patient in the room farthest from the nurses' station.

ANS: A Comfort measures such as washing the face and hands and providing back rubs improve the quality of stimulation and lessen the chance of sensory deprivation. The patient with sensory deprivation needs meaningful stimuli, and therapeutic massage helps establish a humanistic relationship that the patient is missing. All of the other options do not promote patient-nurse interaction and promote further social isolation.

21. A patient has damage to the cerebellum. Which disorder is most important for the nurse to assess? a. Impaired balance b. Hemiplegia c. Muscle sprain d. Lower extremity paralysis

ANS: A Damage to the cerebellum causes problems with balance, and motor impairment is directly related to the amount of destruction of the motor strip. A stroke can lead to hemiplegia. Direct trauma to the musculoskeletal system results in bruises, contusions, sprains, and fractures. A complete transection of the spinal cord can lead to lower extremity paralysis.

9. The nurse is providing discharge education to the patient with diabetes regarding foot care. Which statement by the patient indicates a need for further education? a. "I can go barefoot outside only in the summer." b. "I should wear good fitting shoes." c. "I cannot soak my feet in a hot tub." d. "I can use lotion on my feet."

ANS: A Diabetic patients should not go barefoot outside even in the summer as they often have neuropathy, which decreases the patient's ability to discern touch, especially in the lower extremities. This can lead to foot injuries that can become infected and are slow to heal. The patient should wear good fitting shoes, should avoid extreme temperatures, and can use lotion to keep their skin moist to avoid overly dry skin.

26. The patient is being admitted to the neurological unit with a diagnosis of stroke. When will the nurse begin discharge planning? a. At the time of admission b. The day before the patient is to be discharged c. When outpatient therapy will no longer be needed d. As soon as the patient's discharge destination is known

ANS: A Discharge planning begins when a patient enters the health care system. In anticipation of the patient's discharge from an institution, the nurse makes appropriate referrals or consults a case manager or a discharge planner to ensure that the patient's needs are met at home. Referrals to home care or outpatient therapy are often needed. Planning the day before discharge, when outpatient therapy is no longer needed, and as soon as the discharge destination is known is too late.

29. A nurse is caring for a patient with a right hemisphere stroke and partial paralysis. Which action by an assistive personnel (NAP) demonstrates understanding of the needs of this patient? a. Dressing the left side first b. Dressing the right side first c. Dressing the lower extremities first d. Dressing the upper extremities first

ANS: A Dressing the left side first will be reinforced by the nurse. If a patient has partial paralysis and reduced sensation, the patient dresses the affected side first, in this case, the left. A stroke on the right hemisphere affects the left side of the body. The right side or upper and lower extremities are not as effective.

19. The nurse is ambulating a patient back from the bathroom when the patient begins to have a seizure. Which action should the nurse do first? a. Lower the patient to the floor if standing. b. Move sharp or hard objects away from the patient. c. Turn the patient's head to the side to prevent aspiration. d. Attempt to place a tongue blade to prevent choking.

ANS: A During a seizure, a patient should be protected from injury by first lowering the patient to the ground if standing. The nurse should then place the head on a soft surface and turn it to the side to prevent aspiration and move sharp or hard objects out of the way. The nurse should never attempt to force any object into a seizing patient's mouth.

2. The nurse is caring for an older-adult patient admitted with nausea, vomiting, and diarrhea due to food poisoning. The nurse completes the health history. Which priority concern will require collaboration with social services to address the patient's health care needs? a. The electricity was turned off 3 days ago. b. The water comes from the county water supply. c. A son and family recently moved into the home. d. This home is not furnished with a microwave oven.

ANS: A Electricity is needed for refrigeration of food, and lack of electricity could have contributed to the nausea, vomiting, and diarrhea due to food poisoning. This discussion about the patient's electrical needs can be referred to social services. Foods that are inadequately prepared or stored or subject to unsanitary conditions increase the patient's risk for infections and food poisoning, and an assessment should include storage practices. The water supply, the increased number of individuals in the home, and not having a microwave may or may not be concerns but do not pertain to the current health care needs of this patient.

2. The nurse is assessing a patient's functional performance. What assessment parameters will be most important in this assessment? a. Continence assessment, gait assessment, feeding assessment, dressing assessment, transfer assessment b. Height, weight, body mass index (BMI), vital signs assessment c. Sleep assessment, energy assessment, memory assessment, concentration assessment d. Health and well-being, amount of community volunteer time, working outside the home, and ability to care for family and house

ANS: A Functional impairment, disability, or handicap refers to varying degrees of an individual's inability to perform the tasks required to complete normal life activities without assistance. Height, weight, BMI, and vital signs are part of a physical assessment. Sleep, energy, memory, and concentration are part of a depression screening. Healthy, volunteering, working, and caring for family and house are functional abilities, not performance.

14. A nurse is caring for a patient who is undergoing chemotherapy for cancer. The patient is becoming malnourished because nothing tastes good. Which recommendation by the nurse will be most appropriate for this patient? a. ―Rinse your mouth several times a day to hydrate your taste buds.‖ b. ―Avoid adding spices or lemon juice to food to prevent nausea.‖ c. ―Blend foods together in interesting flavor combinations.‖ d. ―Eat soft foods that are easy to chew and swallow.‖

ANS: A Good oral hygiene keeps the taste buds well hydrated. Having an unpleasant taste in the mouth discourages the patient from eating. Well-seasoned, differently textured food eaten separately heightens taste perception. Avoid blending foods together because this makes it difficult to identify tastes. Texturized, spicy, and aromatic foods stimulate and make eating more enjoyable. Flavored vinegar or lemon juice adds tartness to food.

11. A nurse is teaching a community group of school-aged parents about safety. The proper fitting of which safety item is most important for the nurse to include in the teaching session? a. A bicycle helmet b. Soccer shin guards c. Swimming goggles d. Baseball sliding shorts

ANS: A Head injuries are a major cause of death, with bicycle accidents being one of the major causes of such injuries. Proper fit of the helmet helps to decrease head injuries resulting from these bicycle accidents. Goggles, shin guards, and sliding shorts are important sports safety equipment and should fit properly, but they do not protect from this leading cause of death.

2. A nurse is discharging a patient and is planning on what material to give the patient to take home. What action by the nurse is best? a. Assess the patient's ability to read and understand. b. Determine if the patient wants to take written material home. c. Give the patient the same material as other patients get. d. Ask the patient if he/she has a need for written material.

ANS: A Health literacy in an important concept in health. If the patient cannot read or comprehend written material, it will be of limited use. The nurse first assesses the patient's ability to read and comprehend written material before choosing the material with which to send him/her home. Patients may or may not realize what they need for discharge, if anything. Giving the patient the same material other patients get does not acknowledge their need for holistic and individualized care.

25. A patient has an ankle restraint applied. Upon assessment the nurse finds the toes a light blue color. Which action will the nurse take next? a. Remove the restraint. b. Place a blanket over the feet. c. Immediately do a complete head-to-toe neurological assessment. d. Take the patient's blood pressure, pulse, temperature, and respiratory rate.

ANS: A If the patient has altered neurovascular status of an extremity such as cyanosis, pallor, and coldness of skin or complains of tingling, pain, or numbness, remove the restraint immediately and notify the health care provider. Light blue is cyanosis, indicating the restraints are too tight, not that the patient is cold and needs a blanket. A complete head-to-toe neurological assessment is not needed at this time. The nurse can take vital signs after the restraint is removed.

8. A staff nurse reports a medication error due to failure to administer a medication at the scheduled time. What is the charge nurse's best response? a. "We'll conduct a root cause analysis." b. "That means you'll have to do continuing education." c. "Why did you let that happen?" d. "You'll need to tell the patient and family."

ANS: A In a just culture the nurse is accountable for their actions and practice, but people are not punished for flawed systems. Through a strategy such as root cause analysis the reasons for errors in medication administration can be identified and strategies developed to minimize future occurrences. Requiring continued education may be an appropriate recommendation but not until data is collected about the event. Telling the patient is part of transparency and the sharing and disclosure among stakeholders, but it is generally the role of risk management staff, not the staff nurse.

36. A nurse delegates a position change to a nursing assistive personnel. The nurse instructs the assistive personnel (AP) to place the patient in the lateral position. Which finding by the nurse indicates a correct outcome? a. Patient is lying on side. b. Patient is lying on back. c. Patient is lying semi-prone. d. Patient is lying on abdomen.

ANS: A In the side-lying (or lateral) position, the patient rests on the side with the major portion of body weight on the dependent hip and shoulder. Patients in the supine position rest on their backs. Sims' position is semi-prone. The patient in the prone position lies face or chest down on the abdomen.

5. According to the Health Belief Model, which of the following patients would be most likely to change health behavior? a. The person who perceives that he is at risk for colon cancer b. The person who recognizes that colon cancer is easily cured c. The person who believes that behavior can change outcomes d. The patient who faces multiple social barriers

ANS: A In the three primary components of the Health Belief Model, six main constructs influence an individual's decision to take action about disease prevention, screening, and controlling illness. The model suggests that individuals are motivated to take action if they believe that they are susceptible to the condition (i.e., perceived susceptibility), that the condition has serious consequences (i.e., perceived severity), that taking action would reduce the susceptibility or severity of the condition (i.e., perceived benefit), that the costs of taking action (i.e., perceived barriers) are outweighed by the benefits, that those who are exposed to factors (e.g., media campaigns, postcard reminders, and advice from others) will be prompted to action (i.e., cues to action), and that those who have confidence in their ability to perform an action will do so (i.e., perceived self-efficacy).

3. The nurse is visiting a patient with cardiac disease who has been experiencing increased episodes of shortness of breath when exercise is attempted. The nurse is concerned that the patient's decrease in activity may lead to which outcome? a. Orthostatic hypotension b. Increase risk of heart disease c. Loss of short-term memory d. Worsening shortness of breath

ANS: A Inactivity in patients with cardiopulmonary disease can lead to an unsafe drop in blood pressure with position changes, or orthostatic hypotension. The patient already has heart disease. Loss of short-term memory is not related to the shortness of breath. The lack of activity is not likely to worsen the shortness of breath; improving activity level may help things eventually.

6. A nurse is assessing internal variables that are affecting the patient's health status. Which area should the nurse assess? a. Perception of functioning b. Socioeconomic factors c. Cultural background d. Family practices

ANS: A Internal variables include a person's developmental stage, intellectual background, perception of functioning, and emotional and spiritual factors. External variables influencing a person's health beliefs and practices include family practices, socioeconomic factors, and cultural background.

25. The nurse is caring for a patient who is recovering from a traumatic brain injury and frequently becomes disoriented to everything except location. Which nursing intervention will the nurse add to the care plan to reduce confusion? a. Keep a day-by-day calendar at the patient's bedside. b. Place a patient observer in the patient's room for safety. c. Assess the patient's level of consciousness and document every 4 hours. d. Prepare to discharge once the patient is awake, alert, and oriented.

ANS: A Keeping a calendar in the patient's room helps to orient the patient to the dates. In the home meaningful stimuli include pets, music, television, pictures of family members, and a calendar and clock. The same stimuli need to be present in health care settings. Assessing the patient's level of consciousness is not an action that will directly affect the patient's confusion. A patient observer is unnecessary unless the patient is in danger from the confusion. The nurse should encourage the patient toward recovery but should be sensitive to the time it takes for progression.

13. The nurse is discussing threats to adult safety with a college group. Which statement by a group member indicates understanding of the topic? a. ―Smoking just to control stress is not good for my body.‖ b. ―Our campus is safe; we leave our dorms unlocked all the time.‖ c. ―As long as I have only two drinks, I can still be the designated driver.‖ d. ―I am young, so I can work nights and go to school with 2 hours' sleep.‖

ANS: A Lifestyle choices frequently affect adult safety. Smoking conveys great risk for pulmonary and cardiovascular disease. It is prudent to secure belongings. When an individual has been determined to be the designated driver, that individual does not consume alcohol, beer, or wine. Sleep is important no matter the age of the individual and is important for rest and integration of learning.

2. Several models exist that describe the relationship between health and wellness. Which model is used to understand the interrelationship between elements of basic requirements for survival and the desires that drive personal growth and development and is represented as a pyramid? a. Maslow's hierarchy of needs b. Health Belief Model c. Health Promotion Model d. Holistic Health Model

ANS: A Maslow's hierarchy of needs describes the relationships between the basic requirements for survival and the desires that drive personal growth and development. The model is most often presented as a pyramid consisting of five levels. The lowest level is related to physiologic needs, and the uppermost level is associated with self-actualization needs, specifically those related to purpose and identity. The Health Belief Model was developed by psychologists Hochbaum, Rosenstock, and Kegels. It explores how patients' attitudes and beliefs predict health behavior. The Health Promotion Model, developed by Pender and colleagues, defines health as a positive, dynamic state of well-being rather than the absence of disease in the physiologic state. Holistic health models in nursing care are based on the philosophy that a synergistic relationship exists between the body and the environment. Holistic care is an approach to applying healing therapies. Holistic models focus on the interrelatedness of body and mind.

5. A nurse is caring for an older adult. Which sensory change will the nurse identify as normal during the assessment? a. Impaired night vision b. Difficulty hearing low pitch c. Heightened sense of smell d. Increased taste discrimination

ANS: A Night vision becomes impaired as physiological changes in the aging eye occur. Older adults lose the ability to distinguish high-pitched noises and consonants. Senses of smell and taste are also decreased with aging.

7. The nurse caring for a patient with chronic pain uses guided imagery, therapeutic touch, and relaxation techniques as interventions for pain. The nurse is using what type of approach? a. Holistic b. Eastern holistic c. Risk factor reduction d. Health protection

ANS: A Nurses participate in holistic care through the use of natural healing remedies and complementary interventions. These include the use of art and guided imagery, therapeutic touch, music therapy, relaxation techniques, and reminiscence. Eastern holistic therapists have been using techniques such as acupuncture, yoga, and tai chi for thousands of years as methods of healing and, more recently, in conjunction with modern allopathic medical therapies. Risk factor reduction is step-by-step improvement of individual health factors. These combined improvements lower the likelihood of developing a disease. Health protection includes intentional behaviors aimed at circumventing illness, detecting it early, and maintaining the best possible level of mental and physiologic function within the boundaries of illness.

12. The nurse is caring for an older-adult patient who has been diagnosed with a stroke. Which intervention will the nurse add to the care plan? a. Encourage the patient to perform as many self-care activities as possible. b. Provide a complete bed bath to promote patient comfort. c. Coordinate with occupational therapy for gait training. d. Place the patient on bed rest to prevent fatigue.

ANS: A Nurses should encourage the older-adult patient to perform as many self-care activities as possible, thereby maintaining the highest level of mobility. Sometimes nurses inadvertently contribute to a patient's immobility by providing unnecessary help with activities such as bathing and transferring. Placing the patient on bed rest without sufficient ambulation leads to loss of mobility and functional decline, resulting in weakness, fatigue, and increased risk for falls. After a stroke or brain attack, a patient likely receives gait training from a physical therapist; speech rehabilitation from a speech therapist; and help from an occupational therapist for ADLs such as dressing, bathing and toileting, or household chores.

30. The nurse enters the patient's room and notices a small fire in the headlight above the patient's bed. In which order will the nurse perform the steps, beginning with the first one? 1. Pull the alarm. 2. Remove the patient. 3. Use the fire extinguisher. 4. Close doors and windows. a. 2, 1, 4, 3 b. 1, 2, 4, 3 c. 1, 2, 3, 4 d. 2, 1, 3, 4

ANS: A Nurses use the mnemonic RACE to set priorities in case of fire. The steps are as follows: rescue and remove all patients in immediate danger; activate the alarm; confine the fire by closing doors and windows; and extinguish the fire using an appropriate extinguisher.

9. The nurse recognizes the nursing goal for individuals and families seeking preventative care is to have those groups carry out which action? a. Take responsibility for their health and wellness. b. Abandon the use of electronic educational media. c. Make lifestyle changes after diseases occur. d. Use temporary changes until the danger has passed.

ANS: A Nursing goals for all individuals and their families seeking preventive care are improvement of quality of life through positive lifestyle choices and taking responsibility for health and wellness. Nurses can refer patients to a variety of personal health quizzes, located in the online version of Healthy People 2020, for risk assessments of their health status and lifestyle. The quizzes allow people to track their health and wellness status over a period of years and identify trends in disease risk factors that can be modified through lifestyle interventions or preventive measures before the disease occurs. The Healthy People 2020 initiative helps nurses provide educational materials for individuals, families, and communities, enabling them to lead healthier lifestyles and to make permanent changes in wellness habits.

5. The nurse manager is developing a training guide and identifies which organization that is the best for resources to help develop guidelines to prevent exposure to hazardous situations and decrease the risk of injury in the workplace? a. OSHA (Occupational Safety and Health Administration) b. CDC (Centers for Disease Control and Prevention) c. QSEN (Quality and Safety Education for Nurses) d. NIOSH (National Institute for Occupational Safety and Health)

ANS: A Occupational Safety and Health Administration (OSHA) was established in 1970 to provide employers with guidelines for preventing exposure to hazardous chemicals and hazardous situations and reducing the risk of injury in the workplace. The CDC is the Centers for Disease Control and Prevention and provides information to address exposure to infectious diseases. QSEN, or the Quality and Safety Education for Nurses, was funded by the RWJ to focus on preparing nurses of the future with the knowledge, skills, and attitudes to advance quality and safety on the job. NIOSH, or the National Institute for Occupational Safety and Health, is a federal agency within the CDC that was established to conduct research and recommend interventions for the prevention of work-related injury and illness.

5. A nurse is teaching the patient and family about wound care. Which technique will the nurse teach to best prevent transmission of pathogens? a. Effective hand hygiene b. Saline wound irrigation c. Appropriate use of gloves d. When eye protection is needed

ANS: A One of the most effective methods for limiting the transmission of pathogens is the medically aseptic practice of hand hygiene. The most common means of transmission of pathogens is by the hands. While washing the wound is needed, the best method to prevent transmission is hand hygiene. Wearing gloves and possibly eye protection help protect the nurse, but handwashing is best for limiting the transmission of pathogens.

17. When discussing immunizations for infants and children with new parents, the nurse should focus on which approach? a. Providing scientific evidence to parents b. Stressing that nonimmunization is a crime c. Acknowledging that immunizations are not needed d. Informing the parents that they have no choice

ANS: A Parents need to have scientific, evidence-based information about immunizations and their consequences before choosing to accept or reject immunizations for their children. The parent's ability to make an informed decision is the primary goal for nurses educating people about childhood immunizations.

5. The nurse and a student nurse are discussing the effects of bed immobility on patients. The nurse knows that the student nurse understands the concept of mobility when making which statement? a. "Patients with impaired bed mobility have an increased risk for pressure ulcers." b. "Patients with impaired bed mobility like to have extra visitors." c. "Patients with impaired bed mobility need to have a mechanical soft diet." d. "Patients with impaired bed mobility are prone to constipation."

ANS: A Patients who cannot move themselves in bed are more susceptible to pressure ulcers because they cannot relieve the pressure they feel. Extra visitors or diet consistency do not have any bearing on mobility. Constipation should not be a by-product of immobility if a bowel regimen is instituted.

32. The nurse is caring for a patient who needs to be placed in the prone position. Which action will the nurse take? a. Place pillow under the patient's lower legs. b. Turn head toward one side with large, soft pillow. c. Position legs flat against bed. d. Raise head of bed to 45 degrees.

ANS: A Placing a pillow under the lower leg permits dorsiflexion of the ankles and some knee flexion, which promote relaxation. Head is turned toward one side with a small pillow to reduce flexion or hyperextension of cervical vertebrae. Legs should be supported with pillows to elevate toes and prevent footdrop. Forty-five degrees is the position for Fowler's position; prone is on the stomach.

32. A home health nurse is assessing a family's home after the birth of an infant. A toddler also lives in the home. Which finding will cause the nurse to follow up? a. Plastic grocery bags are neatly stored under the counter. b. Electric outlets are covered in all rooms. c. No bumper pads are in the crib. d. Crib slats are 5 cm apart.

ANS: A Plastic grocery bags increase the risk for suffocation. The nurse will follow up with instructions to remove or keep locked or out of reach. All the rest are correct and do not require follow-up. Electrical outlets should be covered to reduce electrical shock. Bumper pads are not used in the crib to prevent suffocation, strangulation, or entrapment. Crib slats should be less than 6 cm apart.

9. The nurse is working in a clinic that is designed to provide health education and immunizations. Which type of preventive care is the nurse providing? a. Primary prevention b. Secondary prevention c. Tertiary prevention d. Risk factor prevention

ANS: A Primary prevention precedes disease or dysfunction and is applied to people considered physically and emotionally healthy. Primary prevention includes health education programs, immunizations, and physical and nutritional fitness activities. Secondary prevention focuses on individuals who are experiencing health problems or illnesses and who are at risk for developing complications or worsening conditions. Activities are directed at diagnosis and prompt intervention. Tertiary prevention occurs when a defect or disability is permanent and irreversible. It involves minimizing the effects of long-term disease or disability through interventions directed at preventing complications and deterioration. While risk factor modification is an integral component of health promotion, it is not a type of preventive care.

18. The staff nurse knows that many health care facilities use the fire emergency response defined by which acronym? a. RACE b. PASS c. PACE d. QSEN

ANS: A RACE stands for rescue, alarm, contain, and extinguish. QSEN is the Quality and Safety Education for Nurses. PASS is pull, aim, squeeze, and sweep for fire extinguishers. PACE is not a health care acronym.

8. An overweight, sedentary middle-aged smoker with a family history of cardiac disease has noticed a steady rise in resting blood pressure over a 3- to 4-year period. The patient is concerned about his slightly elevated blood pressure and begins walking 20 to 30 minutes in the evenings with his wife and reduces his pack-a-day cigarette habit to ten cigarettes a day. The nurse identifies these actions are the initial step of which behavior? a. Risk factor reduction b. Self-actualization c. Self-transcendence d. Health promotion

ANS: A Risk factor reduction is step-by-step improvement of individual health factors. These combined improvements lower the likelihood of developing a disease. Maslow considered self-actualization the highest level of optimal functioning and involves the integration of cognition, consciousness, and physiologic utility in a single entity. In later years, Maslow described a level above self-actualization called self-transcendence. He refers to self-transcendence as a peak experience, in which analysis of reality or thought changes a person's view of the world and his/her position in the greater structure of life. Health promotion is behavior motivated by the desire to increase well-being (as opposed to preventing illness) and optimize health status.

12. The nurse is presenting an educational session on safety for parents of adolescents. Which information will the nurse include in the teaching session? a. Frequent injuries related to poor psychomotor coordination b. Recognizing common signs and symptoms of the schizophrenia c. Failing grades and changes in dress may indicate substance abuse d. The importance of the use seat belts whenever riding in the backseat of a car

ANS: A School-age children are often uncoordinated. Failing grades and changes in dress may indicate substance abuse. Schizophrenia is not generally noted in adolescents. Seat belts should be used all the time. In fact, teens have the lowest rate of seat belt use.

9. The nurse knows that which patient has a teaching need based on statements by the patient's parents? a. "My 6-month-old daughter only sleeps with me when she's ill." b. "I do not put pillows in the bed with my 3-month-old son." c. "I do not feed popcorn to my 2-year-old." d. "I have discussed the risks of the 'choking game' with my 16-year-old."

ANS: A Small children should never sleep in the bed with others because of the risk of suffocation. The rest of the statements are appropriate. Pillows do present a hazard to a 3-month-old, and popcorn is a choking risk for a 2-year-old. The choking game is a risk to any adolescent.

27. The nurse is caring for a patient who is taking gentamicin for an infection. Which assessment is a priority? a. Hearing b. Vision c. Smell d. Taste

ANS: A Some antibiotics (e.g., streptomycin, gentamicin, and tobramycin) are ototoxic and permanently damage the auditory nerve, whereas chloramphenicol sometimes irritates the optic nerve. Smell and taste are not as affected.

10. When planning interventions for a community, what action by the nurse is best? a. Involve community leaders in planning. b. Create a plan of action addressing priorities. c. Determine what resources are available. d. Attempt to find funding for the plan.

ANS: A Stakeholders need to be involved in planning to ensure buy-in from the community. The stakeholders could be community or business leaders. The other actions are important, but if the community leaders are not committed to the plan, the plan is unlikely to work.

6. The nurse is assessing a patient's functional abilities and asks the patient, "How would you rate your ability to prepare a balanced meal?" "How would you rate your ability to balance a checkbook?" "How would you rate your ability to keep track of your appointments?" Which tool would be indicated for the best results of this patient's perception of their abilities? a. Functional Activities Questionnaire (FAQ) b. Mini Mental Status Exam (MMSE) c. 24hFAQ d. Performance-based functional measurement

ANS: A The FAQ is an example of a self-report tool which provides information about the patient's perception of functional ability. The MMSE assesses cognitive impairment. The 24hFAQ is used to assess functional ability in postoperative patients. Performance-based tools involve actual observation of a standardized task, completion of which is judged by objective criteria.

12. The nurse knows that which assessment tool is not used to assess fall risk? a. Glasgow Falls Scale b. Johns Hopkins Hospital Fall Assessment Tool c. Morse Fall Scale d. Hendrich II Fall Risk Model

ANS: A The Glasgow is a coma scale used to measure level of consciousness, not falls. The rest are scales used to assess the risk for falls in patients.

12. A nurse is assessing cognitive functioning of a patient. Which action will the nurse take? a. Administer a Mini-Mental State Examination (MMSE). b. Ask the patient to state name, location, and what month it is. c. Ask the patient's family if the patient is behaving normally. d. Administer the hearing handicap inventory for the elderly (HHIE-S).

ANS: A The MMSE is a formal diagnostic tool that is used to assess a patient's level of cognitive functioning. The Mini-Mental State Examination (MMSE) is a tool you can use to measure disorientation, change in problem-solving abilities, and altered conceptualization and abstract thinking. Asking the patient orientation questions evaluates only the patient's orientation to self and surroundings, not abstract reasoning or critical thinking ability. Family members are not the most reliable source of information about the patient, although information received from the family should be considered. The HHIE-S is a 5-minute, 10-item questionnaire that assesses how the individual perceives the social and emotional effects of hearing loss. The higher the HHIE-S score, the greater the handicapping effect of a hearing impairment.

6. When there is evidence that supports a screening for an individual patient but not for the general population, the nurse would expect the United States Preventive Services Task Force Grading (USPSTF) to be what? a. No recommendation for or against b. Recommends c. Recommends against d. Strongly recommends

ANS: A The USPSTF Grading is an example of how evidence is used to make guidelines and determine priority. When there is evidence that supports a screening for an individual patient but not for the general population, there is no recommendation for or against screening the general population. Recommends is the grading when there is high certainty that the net benefit is moderate or there is moderate certainty that the net benefit is moderate to substantial. Recommends against is the grading when there is moderate or high certainty that the intervention has no net benefit or that the harms outweigh the benefits. Strongly recommends is the grading when there is high certainty that the net benefit is substantial.

14. The nurse is teaching a group of older adults at an assisted-living facility about age-related physiological changes affecting safety. Which question would be most important for the nurse to ask this group? a. ―Are you able to hear the tornado sirens in your area?‖ b. ―Are you able to read your favorite book?‖ c. ―Are you able to taste spices like before?‖ d. ―Are you able to open a jar of pickles?‖

ANS: A The ability to hear safety alerts and seek shelter is imperative to life safety. Decreased hearing acuity alters the ability to hear emergency vehicle sirens. Natural disasters such as floods, tsunamis, hurricanes, tornadoes, and wildfires are major causes of death and injury. Although age-related changes may cause a decrease in sight that affects reading, and although tasting is impaired and opening jars as arthritis sets in are important to patients and to those caring for them, being able to hear safety alerts is the most important.

18. The nurse recognizes which concept that correctly completes the definition of the genetic vulnerability of an organism (risk of disease expression based on genotype)? a. It is involuntarily passed from biologic parents to offspring. b. It is totally unrelated to environmental factors. c. It is nonresponsive to alteration by way of lifestyle modification. d. It is not a factor in mental illness because it is behavioral.

ANS: A The genetic vulnerability of an organism, or risk of disease expression based on genotype, is involuntarily passed from biologic parents to their offspring. Societal attitudes about testing and management of high-risk populations depend on the potential for expression of genetic disorders that may be triggered by environmental factors. Controlling factors that place stress on physiologic function can reduce pathologic genetic expression and susceptibility to disease. For example, a person with a family history of hyperlipidemia and atherosclerosis is at risk for developing cardiovascular disease later in life. Lifestyle-modifying factors, such as weight reduction, daily exercise, and balanced nutritional intake, can help reduce the likelihood that the genetic risk factor for heart disease will be expressed. Diabetes, cancer, mental illness, and renal disease also have genetic components and are amenable to interventions that reduce risk.

13. Which patient is demonstrating a refractive error sensory problem? a. A patient who frequently reports the incorrect time from the clock across the room. b. A patient who is having difficulty remembering how to perform familiar tasks. c. A patient who turns the television up as loud as possible. d. A patient who has trouble saying words.

ANS: A The most common visual problem is a refractive error such as nearsightedness. Difficulty remembering how to perform familiar tasks indicates the need to further assess mental and cognitive status. Turning the television up louder indicates the need for a hearing assessment. For a patient having trouble saying words a picture board/chart may be used.

24. A nurse is developing an individualized plan of care for a patient. Which action is important for the nurse to take? a. Establish goals that are measurable and realistic. b. Set goals that are a little beyond the capabilities of the patient. c. Use the nurse's own judgment and not be swayed by family desires. d. Explain that without taking alignment risks, there can be no progress.

ANS: A The nurse must develop an individualized plan of care for each nursing diagnosis and must set goals that are individualized, realistic, and measurable. The nurse should set realistic expectations for care and should include the patient and family when possible. The goals focus on preventing problems or risks to body alignment and mobility.

26. A nurse establishing a relationship with the patient who is severely visually impaired is teaching the patient how to contact the nurse for assistance. Which action will the nurse take? a. Place a raised Braille sticker on the call button. b. Explain to the patient that a staff person will stop by once an hour to see if the patient needs anything. c. Instruct the patient to tell a family member to get the attention of the staff. d. Color code the nurse call system.

ANS: A The nurse should devise a plan of care that is accommodating of the patient's visual deficit. Placing a sticker on the nurse call system allows the patient to page the nurse for assistance as needed. Using family members is not the best option. Making hourly rounds is not sufficient; the nurse needs to ensure that the patient can get in touch at any time. Color coding the nurse call system will not help a severely visually impaired patient.

14. The nurse is working in a drug rehabilitation clinic and is in the process of admitting a patient for ―detox.‖ What should the nurse do next? a. Identify the patient's stage of change. b. Realize that the patient is ready to change. c. Teach the patient that choices will have to change. d. Instruct the patient that relapses will not be tolerated.

ANS: A The nurse should identify the stage of change and assess where the patient is currently in this situation. To be most effective, nursing interventions should match the stage of change. The nurse cannot realize the patient is ready for change because only a minority of people are actually in the action stage of changing. While teaching that choices will have to change, it will follow later after the nurse has determined which stage the person is in. As individuals attempt a change in behavior, relapse followed by recycling through the stages occurs frequently.

12. A nurse has assessed a community and has found many areas in which health can be improved. As a result, the nurse has multiple ideas for programming. What action by the nurse is best? a. Determine what the community thinks is most important. b. Use vital statistics to determine which is most important. c. See what other communities are focusing programming on. d. Choose the easiest problem to address first.

ANS: A The nurse's priorities may be very different from the community's. For programming to be successful, there must be buy-in from members of the community. Unless programming addresses a need the community thinks is important, it is unlikely to be successful.

28. A confused patient is restless and continues to try to remove the oxygen cannula and urinary catheter. What is the priority nursing diagnosis and intervention to implement for this patient? a. Risk for injury: Check on patient every 15 minutes. b. Risk for suffocation: Place ―Oxygen in Use‖ sign on door. c. Disturbed body image: Encourage patient to express concerns about body. d. Deficient knowledge: Explain the purpose of oxygen therapy and the urinary catheter.

ANS: A The priority nursing diagnosis is Risk for injury. This patient could cause harm to self by interrupting the oxygen therapy or by damaging the urethra by pulling the urinary catheter out. Before restraining a patient, it is important to implement and exhaust alternatives to restraint. Alternatives can include more frequent observations. This patient may have deficient knowledge; educating the patient about treatments could be considered as an alternative to restraints. However, the nursing diagnosis of highest priority is risk for injury. This scenario does not indicate that the patient has a disturbed body image or that the patient is at risk for suffocation.

15. A nurse is wondering if home health care nursing is a good fit. What characteristic or ability does the experienced home health care nurse suggest is most important? a. Clinical reasoning b. Organization c. Assessment skills d. Time management

ANS: A The role of the registered nurse in home health care is essentially autonomous in that the nurse must be highly proficient in health assessment (physical and psychosocial), be well versed in complex technical and clinical skills, possess strong critical-thinking and clinical reasoning abilities, and demonstrate excellent organizational skills. All choices are important characteristics or abilities of home health care nurses. However, since the nurse working out in the community may not have the resources (personnel or materiel) available in an acute care facility and often must improvise, clinical reasoning would be the most important of the choices provided.

19. A nurse is teaching about the transtheoretical model of change. In which order will the nurse place the progression of the stages from beginning to end? 1. Action 2. Preparation 3. Maintenance 4. Contemplation 5. Precontemplation a. 5, 4, 2, 1, 3 b. 2, 5, 4, 3, 1 c. 4, 5, 3, 1, 2 d. 1, 5, 2, 3, 4

ANS: A The stages of change in the transtheoretical model of change include five stages. These stages range from no intention to change (precontemplation), considering a change within the next 6 months (contemplation), making small changes (preparation), and actively engaging in strategies to change behavior (action), to maintaining a changed behavior (maintenance stage).

1. A nurse is assessing body alignment. What is the nurse monitoring? a. The relationship of one body part to another while in different positions b. The coordinated efforts of the musculoskeletal and nervous systems c. The force that occurs in a direction to oppose movement d. The inability to move about freely

ANS: A The terms body alignment and posture are similar and refer to the positioning of the joints, tendons, ligaments, and muscles while standing, sitting, and lying. Body alignment means that the individual's center of gravity is stable. Body mechanics is a term used to describe the coordinated efforts of the musculoskeletal and nervous systems. Friction is a force that occurs in a direction to oppose movement. Immobility is the inability to move about freely.

17. The nurse is caring for a patient with conductive hearing loss resulting from prolonged cerumen impaction. Which intervention by the nurse is most important in establishing effective communication with the patient? a. Speaking with hands, face, and expressions b. Using a loud voice, enunciating every syllable c. Having direct conversation with the patient in the affected ear d. Repeating the phrase again if the patient does not understand what the nurse said

ANS: A Use visible expressions. Speak with your hands, your face, and your eyes. Do not shout. Speaking in loud tones can distort a patient's ability to hear; the nurse should speak in normal low tones. If the patient does not understand the first time, try rephrasing instead of repeating the message. The nurse can direct conversation toward the patient's unaffected ear.

7. The lack of weight bearing leads to what effects on the skeletal system? a. Demineralization, calcium loss b. Thickened bones c. Increased range of motion d. Increased calcium deposition in the bones

ANS: A Weight bearing helps to strengthen the bone. Lack of weight bearing means that the bone is losing minerals and calcium that strengthen it. Thickened bones will not occur with the lack of weight bearing. Range of motion may be decreased with a lack of weight bearing movements.

17. The nurse is caring for a patient who suddenly becomes confused and tries to remove an intravenous (IV) infusion. Which priority action will the nurse take to minimize the patient's risk for injury? a. Assess the patient. b. Gather restraint supplies. c. Try alternatives to restraint. d. Call the health care provider for a restraint order.

ANS: A When a patient becomes suddenly confused, the priority is to assess the patient, to identify the reason for change in behavior, and to try to eliminate the cause. If interventions and alternatives are exhausted, the nurse working with the health care provider may determine the need for restraints.

14. A nurse is assessing the body alignment of a standing patient. Which finding will the nurse report as normal? a. When observed laterally, the spinal curves align in a reversed ―S‖ pattern. b. When observed posteriorly, the hips and shoulders form an ―S‖ pattern. c. The arms should be crossed over the chest or in the lap. d. The feet should be close together with toes pointed out.

ANS: A When the patient is observed laterally, the head is erect, and the spinal curves are aligned in a reversed ―S‖ pattern. When observed posteriorly, the shoulders and hips are straight and parallel. The arms hang comfortably at the sides. The feet are slightly apart to achieve a base of support, and the toes are pointed forward.

4. The nurse will be most concerned about the risk of malnutrition for a patient with which sensory deficit? a. Xerostomia b. Dysequilibrium c. Diabetic retinopathy d. Peripheral neuropathy

ANS: A Xerostomia is a decrease in production of saliva; this decreases the ability and desire to eat and can lead to nutritional problems. The other options do not address taste- or nutrition-related concerns. Dysequilibrium is balance. Diabetic retinopathy affects vision. Peripheral neuropathy includes numbness and tingling of the affected areas and stumbling gait.

1. Which recommendations would the nurse identify as appropriate screening guidelines? (Select all that apply.) a. Women ages 21 to 29 should have a Pap test every 3 years. b. Self-breast exams should be addressed with male and female patients. c. Adolescent males should perform monthly self-testicular exams. d. Women ages 30 to 65 should receive Pap tests every 10 years. e. After a total hysterectomy, Pap testing should be more frequent.

ANS: A, B, C All women should begin cervical cancer screening at the age of 21 years. Women between the ages of 21 and 29 years should have a Papanicolaou (Pap) test every 3 years. A priority assessment task for nurses in a variety of care settings is to ask female and male patients about breast self-examination. An adolescent male should be assessed for testicular self-examination habits, and older males should have an annual prostate examination. Women between the ages of 30 and 65 years should have a Pap test plus a human papillomavirus (HPV) test (i.e., co-testing) every 5 years. Women 65 years of age or older who have had normal results for previous Pap tests should no longer be screened. Women who have had a total hysterectomy (i.e., removal of the uterus and cervix) should not be tested, unless the surgery was done as a treatment for cervical cancer or pre-cancer.

1. The nurse is teaching a patient about ways to decrease risk of bone fractures. Which statements by the patient indicate a good understanding of decreasing this risk? (Select all that apply.) a. "I should do weight-bearing exercises." b. "I should get adequate intake of calcium and vitamin D." c. "I should exercise regularly." d. "I need to do yoga exercises." e. "I wish I could reduce my risk but can't do anything."

ANS: A, B, C Inadequate dietary intake of calcium and vitamin D or impaired calcium metabolism may result in osteoporosis, which increases bone fragility and may lead to fractures. Decreased physical exercise and lack of weight-bearing exercise also contribute to bone fragility, deterioration, and loss of strength. Any type of exercise will help; it does not need to be yoga, but it does need to include weight-bearing exercise.

8. The home health care nurse educates patients on which goals of hospice care? (Select all that apply.) a. Relieve suffering. b. Support the patient and family. c. Provide grief support. d. Keep patients out of the hospital. e. Lower medical expenses.

ANS: A, B, C The goals of hospice care include relief of suffering, supporting the family and patient, and providing grief support after the patient dies. Goals do not include keeping patients out of the hospital or lowering medical costs.

6. The nurse is assessing a patient who reports a previous fall and is using the SPLATT acronym. Which questions will the nurse ask the patient? (Select all that apply.) a. Where did you fall? b. What time did the fall occur? c. What were you doing when you fell? d. What types of injuries occurred after the fall? e. Did you obtain an electronic safety alert device after the fall? f. What are your medical problems that may have caused the fall?

ANS: A, B, C, D Assess previous falls, using the acronym SPLATT: Symptoms at time of fall Previous fall Location of fall Activity at time of fall Time of fall Trauma after fall Medical diagnoses and an alert device are not components of SPLATT.

7. The nurse is caring for a group of medical-surgical patients. The unit has been notified of a fire on an adjacent wing of the hospital. The nurse quickly formulates a plan to keep the patients safe. Which actions will the nurse take? (Select all that apply.) a. Close all doors. b. Note evacuation routes. c. Note oxygen shut offs. d. Move bedridden patients in their bed. e. Wait until the fire department arrives to act. f. Use type B fire extinguishers for electrical fires.

ANS: A, B, C, D Closing all doors helps to contain smoke and fire. Noting the evacuation routes and oxygen shut offs is important in case evacuation is needed. You will move bedridden patients from the scene of a fire by a stretcher, bed, or wheelchair. The nurse cannot wait until the fire department arrives to act. Type C fire extinguishers are used for electrical fires; type B is used for flammable liquids.

7. The nurse explains to the patient that which services will be covered under Medicare? (Select all that apply.) a. Infusion therapy b. Ostomy management c. Renal dialysis d. Chemotherapy e. Grocery shopping

ANS: A, B, C, D Medicare will reimburse for professionally rendered services provided by a licensed health care provider. Grocery shopping would not be covered. If homemaker services are provided to a patient also receiving skilled care, then they too are reimbursed.

4. The nurse knows which items are included in the documentation for a patient on fall precautions? (Select all that apply.) a. History of any falls b. Falls risk assessment scores c. Patient and family education d. Use of assist devices e. Any fall or reported fall

ANS: A, B, C, D, E The nurse should document the general assessment, include the patient's medical history, subjective and objective data, medication review, musculoskeletal status, and history of falls. Falls assessment and reassessment, patient family education and use of assist devices are also documented. Thoroughly document a fall or reported fall.

3. The nurse is providing education to a community group on environmental safety. Which safety measures are effective in improving their environmental safety? (Select all that apply.) a. Use of night-lights throughout the home b. Illumination of stairwells and pathways c. Installation of motion-activated lighting on the exterior of the home d. Application of wax to all floors to increase shine e. Staying indoors when air pollution is high

ANS: A, B, C, E Inadequate lighting presents safety concerns in home, work, community, and health care environments. For an individual to safely and successfully navigate pathways and perform various activities while avoiding potential obstacles and hazards, the environment must be well illuminated. Well-lit, glare-free halls, stairways, rooms, and work spaces help to reduce the risk of tripping, slipping, and falling. Night-lights reduce the risk of injuries to children, guests, and older adults. Lighting the exterior of the house will also reduce the risk of falling. Staying indoors during episodes when air pollution is high can help prevent chronic lung disease. Waxed floors are slippery.

2. A nurse meets the following goals: helps a patient maintain health and helps a patient with an illness. Which factors assist the nurse in achieving these goals? (Select all that apply.) a. Understands the challenges of today's health care system. b. Identifies actual and potential risk factors. c. Has coined the term ―illness behavior.‖ d. Minimizes the effects of illnesses. e. Experiences compassion fatigue.

ANS: A, B, D Nurses are in a unique position to assist patients in achieving and maintaining optimal levels of health. Nurses understand the challenges of today's health care system. Nurses can identify actual and potential risk factors that predispose a person or group to illness. Nurses who understand how patients react to illness can minimize the effects of illness and assist patients and their families in maintaining or returning to the highest level of functioning. Nurses did not coin the phrase ―illness behavior.‖ While nurses can experience compassion fatigue, it does not help in meeting patient goals.

11. The nurse is preparing discharge instructions for a patient who has equilibrium alterations. Which instructions will the nurse include? (Select all that apply.) a. Use grab bars in the tub and/or shower at home. b. Keep rooms well-lit and focus ahead when walking. c. Change positions quickly to avoid dizziness. d. Use a cane or walker for stability. e. Ride in the back seat of the car and look ahead.

ANS: A, B, D The patient experiencing dizziness or vertigo exercises caution when changing positions. The patient suffering from motion sickness needs to ride in the front seat of the car and look far ahead through the car windshield. Keeping rooms well-lit and focusing ahead when walking, using grab bars in the shower and/or tub, and using canes or walkers are all good safety measures. Changing positions quickly may lead to dizziness.

5. The nurse is performing a health history to determine the patient's sensory status. Which questions will be best suited to elicit the information needed? (Select all that apply.) a. "Do you ever lose your balance?" b. "Do you wear glasses?" c. "Do you like to read the newspaper?" d. "Can you feel the difference between hot and cold water?" e. "Do you wear a hearing aid?"

ANS: A, B, D, E Balance, eyesight, hearing, and sensation are all sensory function. Asking if the patient likes the newspaper does not specifically address vision.

2. A nurse is assessing social determinants of health. Which does the nurse include in the assessment? (Select all that apply.) a. Vaccination compliance b. Family structure c. Communication patterns d. Roles for women e. Education

ANS: A, B, D, E Income, education, health literacy, where people live or work, early childhood development, social exclusion, family structure, the status and role of women, and vaccination adherence are just some of the social determinants of health recognized worldwide. Communication patterns often are important to assess in culturally diverse individuals, families, and communities, but this is not considered a social determinant of health care.

1. The nurse is completing an assessment of an older adult and notices some cognitive impairment not normally associated with aging. Which of these alterations would prompt further follow-up? (Select all that apply.) a. The patient does not remember where her son lives. b. The patient is unable to balance her checkbook. c. The patient got lost in a city she never traveled to before. d. The patient often has difficulty remembering words. e. The patient got lost going to her usual grocery store.

ANS: A, B, D, E Symptoms of cognitive impairment include disorientation, loss of language skills, loss of the ability to calculate, poor judgment, and memory loss. If a patient exhibits these symptoms, further investigation is needed. Some decline in cognitive function occurs with aging, such as the ability to navigate easily in new areas.

7. The nurse is caring for a patient who is hospitalized with cognitive impairment and recognizes which interventions will assist the patient in orientation? (Select all that apply.) a. Keep a photo of the family in the room. b. Use a clock on the wall. c. Make sure the room is kept bright and well lit. d. Avoid moving the patient from room to room. e. Have each nurse introduce himself or herself to the patient.

ANS: A, B, D, E The hospitalized patient with cognitive alterations is oriented by use of a clock, a calendar, and statements about the name of the location or name of the hospital. Orientation to person, place, and time is ongoing. Staff members are always identified by name, both verbally and nonverbally (with a name tag). The patient's environment is kept as constant as possible and moving the patient from room to room is avoided. Some familiar objects, such as a family photo, are placed near the patient if the hospital stay is longer than a few days. The environment is kept free of distractions such as loud noises and bright lights. Natural lighting to provide the patient with orientation to time of day can be accomplished by opening blinds or curtains during the day and darkening the room at night.

3. The nurse is caring for a patient who has experienced a recent stroke and is paralyzed on the left side. The patient has no respiratory or cardiac issues but cannot walk. The patient cannot button a shirt and cannot feed self-due to being left-handed and becomes frustrated very easily. The patient has been eating very little and has lost 2 lb. The patient asks the nurse, ―How can I go home like this? I'm not getting better.‖ Which health care team members will the nurse need to consult? (Select all that apply.) a. Dietitian b. Physical therapist c. Respiratory therapist d. Cardiac rehabilitation therapist e. Occupational therapist f. Psychologist

ANS: A, B, E, F Physical therapists are a resource for planning ROM or strengthening exercises, and occupational therapists are a resource for planning ADLs that patients need to modify or relearn. Because of the loss of 2 lb and eating very little, a dietitian will also be helpful. Referral to a mental health advanced practice nurse, a licensed social worker, or a physiologist to assist with coping or other psychosocial issues is also wise. Because the patient exhibits good cardiac and respiratory function, respiratory therapy and cardiac rehabilitation probably are not needed at this time.

1. A home care nurse is inspecting a patient's house for safety issues. Which findings will cause the nurse to address the safety problems? (Select all that apply.) a. Stairway faintly lit b. Bathtub with grab bars c. Scatter rugs in the kitchen d. Absence of smoke alarms e. Low pile carpeting in the living room f. Level thresholds between bathroom and bedroom

ANS: A, C, D Assess the patient's home for common hazards, including the following: (1) loose area rugs and runner placed over carpeting, (2) poor lighting in stairways, and (3) absence of smoke alarms. Because of reduced depth perception, patients can trip on throw rugs, runners, or the edge of stairs. A bathtub with grab bars is safe and does not need to be addressed. Teach patients and family members to keep all flooring in good repair and advise them to use low-pile carpeting. Thresholds between rooms need to be level with the floor.

2. The nurse is caring for a patient with impaired physical mobility. Which potential complications will the nurse monitor for in this patient? (Select all that apply.) a. Foot drop b. Somnolence c. Hypostatic pneumonia d. Impaired skin integrity e. Increased socialization

ANS: A, C, D Immobility leads to complications such as hypostatic pneumonia. Other possible complications include footdrop and impaired skin integrity. Interruptions in the sleep-wake cycle and social isolation are more common complications than somnolence or increased socialization.

1. A home health nurse is assessing the home for fire safety. Which information from the family will cause the nurse to intervene? (Select all that apply.) a. Smoking in bed helps me relax and fall asleep. b. We never leave candles burning when we are gone. c. We use the same space heater my grandparents used. d. We use the RACE method when using the fire extinguisher. e. There is a fire extinguisher in the kitchen and garage workshop.

ANS: A, C, D Incorrect information will cause the nurse to intervene. Accidental home fires typically result from smoking in bed. Advise families to only purchase newer model space heaters that have all of the current safety features. The PASS method is used for fire extinguishers. All the rest are correct and do not require follow-up. Candles should not be left burning when no one is home. Keep a fire extinguisher in the kitchen, near the furnace, and in the garage.

6. A nurse wants to create a community action plan for health problems related to air pollution from a nearby factory. Which stakeholders does the nurse consult as the priority? (Select all that apply.) a. Factory owners b. Stock shareholders c. Community residents d. Local health care providers e. Factory employees

ANS: A, C, D Stakeholders have a significant interest in a topic. The priority stakeholders the nurse would want to consult for this project include the factory owners, community residents, and health care providers. The stockholders would probably not be consulted. The employees could be a significant stakeholder if the action plan affected employment.

3. A nurse is teaching about the goals of Healthy People 2030. Which information should the nurse include in the teaching concerning what leading health indicator (LHI)? (Select all that apply.) a. Food insecurity b. Healthcare costs c. Adolescent obesity d. Suicide prevention e. Illness care

ANS: A, C, D The current publication, Healthy People 2030, promotes a society in which all people live long, healthy lives. Healthy People 2030 identifies leading health indicators (LHIs) (e.g., household food insecurity and hunger; homicides; suicides; children and adolescents with obesity), which are high-priority health issues in the United States. Healthy People sets objectives to help the United States increase its focus on health promotion and disease prevention (instead of illness care). Healthcare costs while impactful are not identified as a Healthy People 30 LHI.

6. The nurse appropriately delegates care of the unit's patients to the properly trained UAP when that UAP is assigned which tasks? (Select all that apply.) a. UAP assigned to reposition the patient. b. UAP assigned to complete the MORSE falls risk scale. c. UAP assigned to provide range-of-motion exercises. d. UAP assigned to ambulate the patient in the hallway. e. UAP assigned to time the patient on a TUG test.

ANS: A, C, D UAPs provide hands-on care for immobilized patients under the direct supervision of registered nurses. Turning and positioning of patients, range-of-motion exercises, transfers, and assistance with ambulation may be delegated to properly trained UAP. UAPs may not assess patients because that is a nursing responsibility. The MORSE falls risk scale is a risk assessment as is the Timed Up and Go (TUG) test.

9. The nurse is caring for a patient with receptive aphasia. Which interventions will assist the nurse in communicating with the patient? (Select all that apply.) a. Use simple phrases. b. Speak louder than usual. c. Stand in front of the patient. d. Use a picture board. e. Be patient and unrushed.

ANS: A, C, D, E A patient with receptive aphasia cannot understand written or spoken language. Using simple phrases and talking either softly or loudly will not assist that patient. The sensory pathways are intact, but the words do not make sense. A picture board could be used by the nurse when assessing needs. As the patient participates in speech therapy, the ability to understand simple phrases may develop. Standing in front of the patient when talking may give non-verbal clues to the message.

4. The nurse is conducting a windshield survey. What items does the nurse assess? (Select all that apply.) a. Types of housing available b. Cars seen in parking lots c. Recreational facilities d. Health care facilities e. Places of worship

ANS: A, C, D, E A windshield survey is a type of community health assessment. The nurse walks or drives through a neighborhood and notes the type of housing available, the presence and condition of recreational facilities, the presence of health care facilities, and places of worship among other items. Types of cars noted in the neighborhood are not one of the assessments.

10. The nurse identifies that knee-high SCD (Sequential Compression Device) sleeves are correctly placed on the patient when which conditions are met? (Select all that apply.) a. Both sleeves are connected to the SCD device. b. Two fingers fit inside when the SCDs are inflated. c. There are no kinks in the tubing. d. The ankle pressure is 55 to 65 mm Hg. e. The cooling control is on.

ANS: A, C, E Proper positioning of the SCD sleeve allows proper fit and application, which decreases the risk of constricting the blood flow or diminishing optimal outcomes. Wrap the sleeve around the leg and fasten it with Velcro straps. Verify that two fingers fit between the leg and the sleeve when the sleeve is not inflated. Connect the sleeves to the device, ensure that there are no kinks in the tubing, and turn on the cooling and set it to 35 to 55 mm Hg.

2. The nurse knows that a patient with a compromised cardiopulmonary system has a diminished capacity for exercise because of which conditions? (Select all that apply.) a. Decreased tissue perfusion b. Loss of sensation c. Hemiparesis d. Diminished respiratory capacity e. Muscle weakness

ANS: A, D Compromised cardiac function, decreased tissue perfusion, and diminished respiratory capacity directly affect a person's ability to perform activities of daily living (ADLs) and exercise. Hemiparesis and loss of sensation are associated with nervous system disorders. Muscle weakness can be from a number of causes.

9. The nurse is providing discharge education for the patient who is going home with a walker. Which statements by the patient indicate a good level of understanding of safety in the home? (Select all that apply.) a. "I need to remove the throw rugs." b. "I should make sure I only take a bath." c. "I cannot use the stairs." d. "I need to place a nonskid mat in front of the kitchen sink." e. "I wish I had two ways of leaving the house."

ANS: A, D To ensure patients do not have hazards that can cause falls at home, the nurse should evaluate where the living quarters are. If the patient has stairs, they need to be able to safely learn how to use the stairs. They need to remove throw rugs that are a trip hazard and place nonskid mats in front of sinks, tubs, and showers. They can shower with a bench or chair in the shower for sitting. Patients need a clear the exit so they can get out of the house quickly in case of an emergency, but do not specifically need two different exits because of the walker.

2. The nurse recognizes which of the following to be a benefit of regular physical exercise? (Select all that apply.) a. Enhances the immune system. b. Decreases bone density. c. Limits joint mobility. d. Improves mental health. e. Helps to prevent type 2 diabetes.

ANS: A, D, E Exercise is essential for the prevention of illness and promotion of wellness. Physical exercise is any bodily activity or movement that enhances or maintains physical fitness levels and overall health. Exercise strengthens muscles, improves cardiovascular performance, hones athletic skills and endurance, and reduces or maintains weight, and it is performed for enjoyment (Powers and Howley, 2012). Regular physical exercise enhances the immune system, builds and maintains healthy bone density, increases joint mobility, and helps to prevent cardiovascular disease, type 2 diabetes, and obesity. Exercise also improves mental health and helps to prevent depression through the release of endorphins and other neurotransmitters that are responsible for exercise-induced euphoria (Powers and Howley, 2012).

1. Upon assessment a nurse discovers that a patient has erythema. Which actions will the nurse take? (Select all that apply.) a. Consult a dietitian. b. Increase fiber in the diet. c. Place on chest physiotherapy. d. Increase frequency of turning. e. Place on pressure-relieving mattress.

ANS: A, D, E If skin shows areas of erythema and breakdown, increase the frequency of turning and repositioning; place the turning schedule above the patient's bed; implement other activities per agency skin care policy or protocol (e.g., assess more frequently, consult dietitian, place patient on pressure-relieving mattress). Increased fiber will help constipation. Chest physiotherapy is for respiratory complications.

4. The nurse is performing a health history to determine the patient's cognitive status. Which questions will be best suited to elicit the information needed? (Select all that apply.) a. "Are you able to drive to the store or do errands?" b. "Do you have any pain?" c. "Is your vision blurry?" d. "Are you able to smell different foods?" e. "Have you noticed any difficulty adding up numbers?"

ANS: A, E Driving and adding numbers relates to cognitive ability. The remaining three options have a sensory focus.

3. A nurse is studying intrinsic factors that influence the development of asthma in a community. What factors does the nurse assess? (Select all that apply.) a. Socioeconomic status b. Genetics c. Pollution in the area d. Water cleanliness e. Immunization status

ANS: A, E Host, or intrinsic factors are individual variables such as genetics, age, gender, ethnic group, immunization status, and human behavior that impact a person's health. The other options are all extrinsic factors, which pertain to environmental characteristics.

3. The nurse is assessing a patient with a mobility dysfunction and wants to gain insight into the patient's functional ability. What question would be the most appropriate? a. "Are you able to shop for yourself?" b. "Do you use a cane, walker, or wheelchair to ambulate?" c. "Do you know what today's date is?" d. "Were you sad or depressed more than once in the last 3 days?"

ANS: B "Do you use a cane, walker, or wheelchair to ambulate?" will assist the nurse in determining the patient's ability to perform self-care activities. A nutritional health risk assessment is not the functional assessment. Knowing the date is part of a mental status exam. Assessing sadness is a question to ask in the depression screening.

36. An older-adult patient is using a wheelchair to attend a physical therapy session. Which action by the nurse indicates safe transport of the patient? a. Positions patient's buttocks close to the front of wheelchair seat. b. Backs wheelchair into elevator, leading with large rear wheels first. c. Places locked wheelchair on same side of bed as patient's weaker side. d. Unlocks wheelchair for easy maneuverability when patient is transferring.

ANS: B A correct action when using a wheelchair is to back wheelchair into an elevator, leading with large rear wheels first. A patient's buttocks should be well back into the seat. A locked wheelchair should be placed on a patient's strong or unaffected side. Brakes should be securely locked when a patient is transferring.

35. The nurse is caring for a patient who is immobile and needs to be turned every 2 hours. Which device will the nurse use to help maintain foot function? a. Hand rolls b. A foot boot c. A trapeze bar d. A trochanter roll

ANS: B A foot boot prevents foot drop by maintaining a foot in dorsiflexion. A trochanter roll prevents external rotation of the hips when the patient is in a supine position. This is especially useful in patients who have lost the ability to move the lower extremities. Hand rolls maintain the thumb in slight adduction and in opposition to the fingers. The trapeze bar is a triangular device that hangs down from a securely fastened overhead bar that is attached to the bedframe. It allows the patient to pull with the upper extremities to raise the trunk off the bed, to assist in transfer from bed to wheelchair, or to perform upper arm exercises.

22. The nurse caring for a group of patients is monitoring for sensory deprivation. Which patient will the nurse monitor most closely? a. A patient in the ICU under constant monitoring following a myocardial infarction b. A patient on the unit with tuberculosis on airborne precautions c. A patient who recently had a stroke and has left-sided weakness d. A patient receiving hospice care for end-stage lung cancer

ANS: B A group at risk includes patients isolated in a health care setting or at home because of conditions such as active tuberculosis. Sensory deprivation occurs when a person has decreased stimulation and limited sensory input. A patient in isolation (airborne precautions) is at risk for sensory deprivation because of limited exposure to meaningful stimuli. A patient in the ICU would be at risk for sensory overload with all the monitors and visitors. A patient with a stroke may have difficulty with tactile sensation but is not at as high a risk for sensory deprivation as is one in isolation. A patient with lung cancer may have deficits, but hospice is present so the patient is at home with others.

34. A home health nurse is teaching a family to prevent electrical shock. Which information will the nurse include in the teaching session? a. Run wires under the carpet. b. Disconnect items before cleaning. c. Grasp the cord when unplugging items. d. Use masking tape to secure cords to the floor.

ANS: B A guideline to prevent electrical shock is to disconnect items before cleaning. Do not run wires under carpeting. Grasp the plug, not the cord, when unplugging items. Use electrical tape to secure the cord to the floor, preferably against baseboards.

7. The nurse is admitting a patient diagnosed with uncontrolled diabetes mellitus. It is the fourth time the patient is being admitted in the last 6 months for high blood glucose levels. During the admission process, the nurse asks the patient about employment status and displays a nonjudgmental attitude. What is the rationale for the nurse's actions? a. External variables have little effect on adherence. b. A person's adherence is affected by economic status. c. Employment status is an internal variable that impacts compliance. d. Noncompliant patients thrive on the disapproval of authority figures.

ANS: B A person's adherence with treatment is affected by economic status. A person tends to give a higher priority to food and shelter than to costly drugs or treatments. External variables can have a major impact on compliance. Employment status is an external variable, not an internal variable. A person generally seeks approval and support from social networks, and this desire for approval affects health beliefs and practices; noncompliance does not occur from thriving on disapproval of authority figures.

9. The nurse correctly selects which intervention to avoid causing shear or friction when moving a patient in bed? a. Using an airflow bed b. Using a slide board c. Using a trochanter roll d. Using a gel mattress

ANS: B A transfer or slide board is made of plastic-like material that reduces friction. Linens easily slide over the board, facilitating bed linen changes. Patients can be repositioned or transferred with a minimum of force required. A trochanter roll prevents outward rolling of the hip when a patient is lying on his/her back. An air-fluidized bed uses airflow to move silicone particles in the bed, creating a watery, fluid-like movement and resulting in lower pressure to avoid or alleviate decubitus ulcers. A foam or gel combination mattress reduces pressure.

35. The nurse has placed a yellow armband on a 70-year-old patient. Which observation by the nurse will indicate the patient has an understanding of this action? a. The patient removes the armband to bathe. b. The patient wears the red nonslip footwear. c. The patient insists on taking a ―water‖ pill in the evening. d. The patient who is allergic to penicillin asks the name of a new medicine.

ANS: B A yellow armband is an alert for high risk of falls. Red nonslip footwear helps to grip the floor and decreases the chance of falling. The communication armband should stay in place and should not be removed, so that all members of the interdisciplinary team have the information about the high risk for falls. A red armband indicates an allergy. Give diuretics (―water‖ pill) in the morning to decrease risk of falls during the night—when most falls occur.

4. A nurse is using Maslow's hierarchy to prioritize care for an anxious patient that is not eating and will not see family members. Which area should the nurse address first? a. Anxiety b. Not eating c. Mental health d. Not seeing family members

ANS: B According to Maslow, in all cases an emergent physiological need takes precedence over a higher-level need. Nutrition is a physiological need and should be addressed first. Anxiety, mental health, and not seeing family members (need for love and belonging) are all higher-level needs.

19. A nurse is working to prevent blindness. Which preventive action is a priority? a. Screen young adults early for visual impairments. b. Include rubella and syphilis screening in the preconception care plan. c. Instruct parents to report reduced eye contact from their child immediately. d. Administer eye prophylactic antibiotics to newborns within 24 hours after birth.

ANS: B Actions to prevent blindness must occur before vision impairment takes place. Screening for diseases such as rubella, syphilis, chlamydia, and gonorrhea that affect development of vision in the fetus is a preventative measure. Vision testing after birth is important to begin steps to correct or identify the problem early on so the child can develop as normally as possible; waiting until children are young adults is too late. Another technique is administering eye prophylaxis in the form of erythromycin ointment approximately 1 hour after an infant's birth. Reporting reduced eye contact is recommended but is not a preventative measure.

10. Which explanation by the nurse best describes active assistive range of motion? a. The patient independently moves all joints. b. The patient to partially moves all joints. c. The caregiver must move the patient's joints. d. The patient performs isotonic exercises.

ANS: B Active assistive range of motion occurs when the caregiver minimally assists the patient, or the patient minimally assists himself/herself in the movement of joints through a full motion. Active range of motion occurs when the patient has full independent movement of all joints; this is also known as isotonic exercise. Passive range of motion occurs when the caregiver moves the patient's joints through a full motion. This exercise does not maintain or improve strength but maintains flexibility and prevents contractures and atrophy.

20. The nurse is caring for a confused, combative patient. Which action would be considered last by the nurse to control behavior of the client? a. Orient the patient frequently. b. Apply restraints. c. Move the patient to a room close to the nurse's station. d. Encourage the family to spend time with the patient.

ANS: B All alternatives to physical restraints should be considered prior to their use.

17. The patient had a colostomy placed 1 week ago. When approached by the nurse, the patient and their spouse refuse to talk about it and reject the opportunity to be taught about how to care for it. How will the nurse evaluate this couple's stage of adjustment? a. Shock b. Withdrawal c. Acceptance d. Rehabilitation

ANS: B As the patient and family recognize the reality of a change, they become anxious and may withdraw, refusing to discuss it. This is an adaptive coping mechanism that assists the patient in making the adjustment. Initially, the patient may be shocked by the change. This is followed by withdrawal, acknowledgment, acceptance, and rehabilitation (ready to adapt to the change through use of colostomy bag).

16. The nurse is assessing body alignment for a patient who is immobilized. Which patient position will the nurse use? a. Supine position b. Lateral position c. Lateral position with positioning supports d. Supine position with no pillow under the patient's head

ANS: B Assess body alignment for a patient who is immobilized or bedridden with the patient in the lateral position, not supine. Remove all positioning supports from the bed except for the pillow under the head and support the body with an adequate mattress.

43. The patient is admitted to a skilled care unit for rehabilitation after the surgical procedure of fixation of a fractured left hip. The patient's nursing diagnosis is Impaired physical mobility related to musculoskeletal impairment from surgery and pain with movement. The patient is able to use a walker but needs assistance ambulating and transferring from the bed to the chair. Which nursing intervention is most appropriate for this patient? a. Obtain assistance and physically transfer the patient to the chair. b. Assist with ambulation and measure how far the patient walks. c. Give pain medication after ambulation so the patient will have a clear mind. d. Bring the patient to the cafeteria for group instruction on ambulation.

ANS: B Assist with walking and measure how far the patient walks to quantify progress. The nurse should allow the patient to do as much for self as possible. Therefore, the nurse should observe the patient transferring from the bed to the chair using the walker and should provide assistance as needed. The patient should be encouraged to use adequate pain medication to decrease the effects of pain and to increase mobility. The patient should be instructed on safe transfer and ambulation techniques in an environment with few distractions, not in the cafeteria.

17. The nurse is assessing the patient for respiratory complications of immobility. Which action will the nurse take when assessing the respiratory system? a. Inspect chest wall movements primarily during the expiratory cycle. b. Auscultate the entire lung region to assess lung sounds. c. Focus auscultation on the upper lung fields. d. Assess the patient at least every 4 hours.

ANS: B Auscultate the entire lung region to identify diminished breath sounds, crackles, or wheezes. Perform a respiratory assessment at least every 2 hours for patients with restricted activity. Inspect chest wall movements during the full inspiratory-expiratory cycle. Focus auscultation on the dependent lung fields because pulmonary secretions tend to collect in these lower regions.

16. Upon completion of the assessment, the nurse finds that the patient has quit drinking and has been alcohol free for the past 2 years. Which stage best describes the nurse's assessment finding? a. Contemplation b. Maintenance c. Preparation d. Action

ANS: B Because the patient has been alcohol free for 2 years, the patient is in the maintenance stage. These stages range from no intention to change (precontemplation), to considering a change within the next 6 months (contemplation), to making small changes (preparation), to actively engaging in strategies to change behavior (action), to maintaining a changed behavior (maintenance).

15. The nurse is evaluating the body alignment of a patient in the sitting position. Which observation by the nurse will indicate a normal finding? a. The edge of the seat is in contact with the popliteal space. b. Both feet are supported on the floor with ankles flexed. c. The body weight is directly on the buttocks only. d. The arms hang comfortably at the sides.

ANS: B Both feet are supported on the floor, and the ankles are comfortably flexed. Body weight is evenly distributed on the buttocks and thighs. A 1- to 2-inch space is maintained between the edge of the seat and the popliteal space on the posterior surface of the knee to ensure that no pressure is placed on the popliteal artery or nerve. The patient's forearms are supported on the armrest, in the lap, or on a table in front of the chair.

13. A home health care nurse is working with the family of a patient who has Alzheimer disease and requires 24-hour care. What assessment by the nurse indicates the family is meeting an important goal for caregiver role stress? a. Family eats dinner together every night. b. Family uses respite care one night a week. c. Family investigates research trials for patient. d. Family verbalizes exhaustion from caregiving.

ANS: B Caregiver role stress can occur when the caregiver(s) is unable to meet obligations or unable to take care of personal needs. Using a respite caregiver once a week gives the family a little time off to accomplish needed tasks. The other observations are not tied to this diagnosis.

4. The nurse is caring for a patient with depression. Which statement by the patient indicates a need for further education? a. "Depression can be caused by chemical changes in the brain." b. "Depression is always treated with medication." c. "Depression is a mood disorder." d. "Depression can have a rapid onset."

ANS: B Depression is usually reversible with treatment either by eliminating the underlying cause, providing counseling, or prescribing antidepressive agents. Depression is a mood disorder and is believed to be caused by chemical changes in the brain. Depression usually has a rapid onset, and the patient's mood is constant.

29. A nurse is caring for a patient diagnosed with osteoporosis and lactose intolerance. What intervention will the nurse implement? a. Encourage dairy products. b. Monitor intake of calcium. c. Increase intake of caffeinated drinks. d. Try to do as much as possible for the patient.

ANS: B Encourage patients at risk to be screened for osteoporosis and assess their diets for calcium and vitamin D intake. Patients who have lactose intolerance need dietary teaching about alternative sources of calcium. Caffeine should be decreased. The goal of the patient with osteoporosis is to maintain independence with ADLs. Assistive ambulatory devices, adaptive clothing, and safety bars help the patient maintain independence.

6. The nurse is educating parents about firearm safety. Which parent statement indicates to the nurse a need for further education? a. "I should make sure I obtain the proper permits." b. "It is okay to store firearms with ammunition loaded." c. "I should store all firearms without ammunition." d. "I should make sure all firearms have trigger locks in place."

ANS: B Firearms should be stored in a secure location with trigger locks in place. Ammunition should be stored in a separate location also locked. Proper permits should be obtained as appropriate. Loaded firearms should never be stored where children can access them.

1. Which tertiary prevention measure should be included in the health promotion plan of care for a patient newly diagnosed with diabetes? a. Avoiding carcinogens b. Foot screening techniques c. Glaucoma screening d. Seat belt use

ANS: B Foot screening is considered a tertiary prevention measure, one that minimizes the problems with foot ulcers, an effect of diabetic disease and disability. Avoiding carcinogens is considered primary prevention—those strategies aimed at optimizing health and disease prevention in general and not linked to a single disease entity. Glaucoma screening is considered secondary screening—measures designed to identify individuals in an early state of a disease process so that prompt treatment can be started. Seat belt use is considered primary prevention—those strategies aimed at optimizing health and disease prevention in general and not linked to a single disease entity.

37. A nurse is evaluating care of an immobilized patient. Which action will the nurse take? a. Focus on whether the interdisciplinary team is satisfied with the care. b. Compare the patient's actual outcomes with the outcomes in the care plan. c. Involve primarily the patient's family and health care team to determine goal achievement. d. Use objective data solely in determining whether interventions have been successful.

ANS: B From your perspective as the nurse, you are to evaluate outcomes and response to nursing care and compare the patient's actual outcomes with the outcomes selected during planning. Ask if the patient's expectations (subjective data) of care are being met and use objective data to determine the success of interventions. Just as it was important to include the patient during the assessment and planning phase of the care plan, it is essential to have the patient's evaluation of the plan of care, not just the patient's family and health care team.

18. The home health nurse is caring for a patient with tactile and visual deficits. The nurse is concerned about injury related to inability to feel harmful stimuli and teaches the patient safety strategies to maintain independence. Which action by the patient indicates successful learning? a. Asks the nurse to test the temperature of the water before entering the bath. b. Places colored stickers on faucet handles to indicate temperature. c. Replaces all lace-up shoes with Velcro straps for ease. d. Uses a heating pad on a low setting to keep warm.

ANS: B If a patient with tactile deficits also has a visual impairment, it is important to be sure that water faucets are clearly marked ―hot‖ and ―cold,‖ or use color codes (i.e., red for hot and blue for cold). Discourage the use of heating pads in this population. Asking the nurse to test the water does not promote independence, although it does promote safety. Velcro is easier for a patient with a tactile deficit to manipulate and promotes self-care but not safety.

2. The nurse is implementing generalized falls precautions for patients who are at risk for falls. Which intervention indicates a lack of understanding of these precautions? a. The bed is placed in the low position. b. The patient is wearing socks. c. The patient's cell phone is by the bedside. d. The patient's call light is within reach.

ANS: B If the patient is ambulatory, require the use of nonskid footwear. Socks can be slippery unless they have a grip surface on them. Keep patient belongings (e.g., tissues, water, urinals, personal items) within the patient's reach. Keep the call light in reach and remind the patient to use it and keep the bed in the low position.

4. At the well-child clinic, how does the nurse correctly teach a mother about health promotion activities and describe immunizations? a. Unique for children b. Primary prevention c. Secondary prevention d. Tertiary prevention

ANS: B Immunizations/vaccinations are considered primary prevention measures, those strategies aimed at optimizing health and disease prevention in general. Immunizations/vaccinations are primary prevention measures for individuals across the lifespan, not just children. Secondary prevention measures are those designed to identify individuals in an early state of a disease process so that prompt treatment can be started. Tertiary prevention measures are those that minimize the effects of disease and disability.

8. When making rounds the nurse observes a purple wristband on a patient's wrist. What information about the patient does this provide the nurse? a. They are allergic to certain medications or foods. b. A ―Do not resuscitate‖ order is in effect. c. The patient has a high risk for falls. d. The patient is at risk for seizures.

ANS: B In 2008 the American Hospital Association issued an advisory recommending that hospitals standardize wristband colors: red for patient allergies, yellow for fall risk, and purple for do not resuscitate preferences. Purple does not indicate seizures.

6. The nurse is providing discharge instructions to an older adult who is being discharged with orthostatic hypotension. Which response by the patient indicates a need for further education? a. "I should take my blood pressure once a day at home." b. "I should get up quickly to avoid my blood pressure dropping." c. "I should drink plenty of water during the day." d. "I should get up slowly and carefully."

ANS: B In orthostatic hypotension, dizziness and loss of consciousness may occur if a patient changes position too quickly. Instead they should change positions slowly. A patient can take their blood pressure at home to monitor it. Drinking water will keep them hydrated.

42. A nurse is assessing pressure points in a patient placed in the Sims' position. Which areas will the nurse observe? a. Chin, elbow, hips b. Ileum, clavicle, humerus c. Shoulder, anterior iliac spine, ankles d. Occipital region of the head, coccyx, heels

ANS: B In the Sims' position, pressure points include the ileum, humerus, clavicle, knees, and ankles. The lateral position pressure points include the ear, shoulder, anterior iliac spine, and ankles. The prone position pressure points include the chin, elbows, female breasts, hips, knees, and toes. Supine position pressure points include the occipital region of the head, vertebrae, coccyx, elbows, and heels.

19. The patient is asking about using the Internet for resources regarding lifestyle behaviors and benefits of modification. What is the best response that the nurse should provide the patient? a. Information on lifestyle behaviors is not available on the Internet. b. The patient should use websites that are easy to understand. c. Most websites are designed for health care providers only. d. Only negative outcomes are evaluated on the Internet.

ANS: B Information on lifestyle behaviors that lead to disease is available at research-sponsored websites that have peer-reviewed material and expert analyses. Website content should be easy to read and understandable for the general population. Most sites that discuss the latest information about health risks, lifestyle behaviors, and outcomes have separate information specifically for health care providers. Research that evaluates positive and negative lifestyle-behavior outcomes is constantly evolving as discoveries are made about the physiologic changes bodies experience with disease and illness.

11. The nurse recognizes which goal to be appropriate for the patient with a Nursing diagnosis of social isolation? a. The patient will participate in cognitive exercises. b. The patient will interact with other residents during activities. c. The patient will communicate basic needs through use of photos. d. The patient will remain within the unit while in long-term care.

ANS: B Interacting with others during activities is an appropriate goal to help the patient not feel so alone. Cognitive exercise is a goal for a patient with disturbed thought processes. Communication of basic needs through the use of photos is a goal for a patient with a diagnosis of impaired verbal communication and remaining in the unit is appropriate for chronic confusion.

26. The emergency department has been notified of a potential bioterrorism attack. Which action by the nurse is priority? a. Monitor for specific symptoms. b. Manage all patients using standard precautions. c. Transport patients quickly and efficiently through the elevators. d. Prepare for posttraumatic stress associated with this bioterrorism attack.

ANS: B Manage all patients with suspected or confirmed bioterrorism-related illnesses using standard precautions. For certain diseases, additional precautions may be necessary. The early signs of a bioterrorism-related illness often include nonspecific symptoms (e.g., nausea, vomiting, diarrhea, skin rash, fever, confusion) that may persist for several days before the onset of more severe disease. Limit the transport and movement of patients to movement that is essential for treatment and care. Psychosocial concerns (posttraumatic stress) are important but are not the first priority at this moment.

12. The nurse knows that manual lifting should only be done in which situation? a. Patients who are less than 150 lb b. Life-threatening situations c. Postsurgical patients d. Patients who are less than 200 lb

ANS: B Many manual patient handling tasks are unsafe, because the weights lifted and movements required are beyond the ability of most caregivers. The key is to identify the task to be accomplished, and then use the required equipment and personnel so that the task fits the capabilities of the staff (U.S. Department of Veterans Affairs, 2016). The patient's level of cooperation is taken into consideration when using the safe patient-handling and mobility (SPHM) algorithms to decide the best method of moving the patient. The patient's weight, medical conditions, and ability to assist are also considered (U.S. Department of Veterans Affairs, 2016). Postsurgical patients as well as patients less than 150 or 200 lb may not fit the criteria.

18. The nurse is assessing an immobile patient for deep vein thromboses (DVTs). Which action will the nurse take? a. Remove elastic stockings every 4 hours. b. Measure the calf circumference of both legs. c. Lightly rub the lower leg for redness and tenderness. d. Dorsiflex the foot while assessing for patient discomfort.

ANS: B Measure bilateral calf circumference and record it daily as an assessment for DVT. Unilateral increases in calf circumference are an early indication of thrombosis. Homan's sign, or calf pain on dorsiflexion of the foot, is no longer a reliable indicator in assessing for DVT, and it is present in other conditions. Remove the patient's elastic stockings and/or sequential compression devices (SCDs) every 8 hours, and observe the calves for redness, warmth, and tenderness. Instruct the family, patient, and all health care personnel not to massage the area because of the danger of dislodging the thrombus.

23. During the admission assessment, the nurse assesses the patient for fall risk. Which finding will alert the nurse to an increased risk for falls? a. The patient is oriented. b. The patient takes a hypnotic. c. The patient walks only 2 miles a day. d. The patient recently became widowed.

ANS: B Numerous factors increase the risk of falls, including a history of falling and the effects of various medications such as anticonvulsants, hypnotics, sedatives, and certain analgesics. Being oriented will decrease risk for falls while disorientation will increase the risk of falling. Walking has many benefits, including increasing strength, which would be beneficial in decreasing risk. Becoming widowed would increase stress and may affect concentration but is not a great risk.

8. A nurse is completing an OASIS assessment on a patient. What data would be most important for the nurse to assess? a. Presence of grocery stores nearby b. Safety concerns within the home c. Number and kind of pets d. Proximity to a health care facility

ANS: B OASIS (Outcomes and Assessment Information Set) is a data set of outcome measures for adult home health care clients that is used to track outcome-based quality improvement. Factors that could potentially affect patient safety in the home are particularly important. The other options are not included in this assessment.

21. A nurse is attempting to minimize the risk of future infection for a post-surgical patient about to be discharged. Which technique will the nurse teach the patient to best achieve this goal? a. Sanitizing of eating utensils b. Medical asepsis handwashing c. Wound care using surgical asepsis d. Limiting visitors during flu season

ANS: B One of the most effective methods for limiting the transmission of pathogens in health care is the medically aseptic practice of hand hygiene. It is important that nurses educate patients, families, and caregivers about the importance of incorporating hand hygiene in all aspects of their lives. While effective dish washing and limiting visitors during periods when the risk for an infection is elevated may be prudent, neither will be as impactful as effective handwashing. Surgical asepsis is needed in only very select situations.

11. A nurse has referred a patient to a community agency. When talking to the patient later, he states that he did not find the agency helpful. What action by the nurse is best? a. Determine what the patient would find helpful. b. Review the agency's mission and scope. c. Make another appointment with the agency. d. Warn the patient that nonadherence affects payment.

ANS: B One of the most important aspects of a community health nurse's role is to be familiar with referral agencies. Awareness of the scope of an agency's influence and services helps the community nurse to pinpoint which agencies are best able to address specific needs. The nurse may have sent this patient to an agency that did not meet his needs. The nurse should ask the patient's opinion about what services are needed. Making another appointment without ensuring that this is the right agency for the patient will not solve the problem. Telling the patient that payment might not be ensured for nonadherence is not therapeutic communication.

20. The nurse is assessing a patient's environment and its impact on outdoor activity and notes that the child rarely plays outside. Which is true regarding the indoor environment? a. Indoor environments protect the patient from toxics chemicals. b. Indoor activity is sometimes a result of unsafe outdoor conditions. c. Indoor activity decreases the risk of respiratory illness. d. Indoor lifestyles reduce the risk for sedentary behaviors.

ANS: B Outdoor environments affect individual health in the areas of sanitation and waste disposal, water quality, air quality, and safety. Children living in areas where there are safety issues related to gang activity, sexual predators, or heavy traffic are less likely to engage in outdoor play activities. Their limited access to safe outdoor play space increases their risk for sedentary behaviors, excessive calorie intake, and obesity. Indoor environments may harbor toxic household cleaning agents, chemicals (e.g., radon, carbon monoxide, unused drugs), tobacco smoke, and energy sources (e.g., microwave ovens). Exposure to mold, household pests (e.g., dust mites, spiders), and unsanitary living conditions in an enclosed space increases the likelihood of respiratory illness and skin disorders.

40. The nurse is caring for a patient who has experienced a stroke causing total paralysis of the right side. To help maintain joint function and minimize the disability from contractures, passive range of motion (ROM) will be initiated. When should the nurse begin this therapy? a. After the acute phase of the disease has passed b. As soon as the ability to move is lost c. Once the patient enters the rehab unit d. When the patient requests it

ANS: B Passive ROM exercises should begin as soon as the patient's ability to move the extremity or joint is lost. The nurse should not wait for the acute phase to end. It may be some time before the patient enters the rehab unit or the patient requests it, and contractures could form by then.

15. The nurse is caring for a hospitalized patient. Which behavior alerts the nurse to consider the temporary need for a restraint? a. The patient refuses to call for help to go to the bathroom. b. The patient continues to remove the nasogastric tube. c. The patient gets confused regarding the time at night. d. The patient does not sleep and continues to ask for items.

ANS: B Patients who are confused, disoriented, and wander or repeatedly fall or try to remove medical devices (e.g., oxygen equipment, IV lines, or dressings) often require the temporary use of restraints to keep them safe. Restraints can be used to prevent interruption of therapy such as traction, IV infusions, NG tube feeding, or Foley catheterization. Refusing to call for help, although unsafe, is not a reason for restraint. Getting confused at night regarding the time or not sleeping and bothering the staff to ask for items is not a reason for restraint.

5. The nurse recognizes which goal to be appropriate for the patient who is postoperative day one from a hip fracture with the nursing diagnosis Impaired mobility? a. Patient will interact with others. b. Patient will ambulate to the bathroom with assistance. c. Patient will have no skin breakdown. d. Patient will have a physical therapy consult.

ANS: B Patients with a diagnosis of Impaired mobility should have a goal aimed at improving their mobility. Although immobility can impact social isolation and skin breakdown, those goals are not appropriate for this diagnosis. Have a physical therapy consult is not a goal but an intervention.

8. A nurse reviews an immobilized patient's laboratory results and discovers hypercalcemia. Which condition will the nurse monitor for most closely in this patient? a. Hypostatic pneumonia b. Renal stones c. Pressure ulcers d. Thrombus formation

ANS: B Renal calculi are calcium stones that lodge in the renal pelvis or pass through the ureters. Immobilized patients are at risk for calculi because they frequently have hypercalcemia. Hypercalcemia does not lead to hypostatic pneumonia, pressure ulcers, or thrombus formation. Immobility is one cause of hypostatic pneumonia, which is inflammation of the lung from stasis or pooling of secretions. A pressure ulcer is an impairment of the skin that results from prolonged ischemia (decreased blood supply) within tissues. A thrombus is an accumulation of platelets, fibrin, clotting factors, and cellular elements of the blood attached to the interior wall of a vein or artery, which sometimes occludes the lumen of the vessel.

7. When the nurse discovers a patient on the floor, the patient states, ―I fell out of bed‖. The nurse assesses the patient and then places the patient back in bed. Which action should the nurse take next? a. Do nothing, no harm has occurred. b. Notify the health care provider. c. Complete an incident report. d. Re-assess the patient.

ANS: B Report immediately to physician or health care provider if the patient sustains a fall or an injury. The nurse must provide safe care and doing nothing is not safe care. The scenario indicates the nurse has already assessed the patient. After the patient has stabilized, completing an incident report would be the last step in the process.

11. The nurse identifies which instruction to be appropriate to delegate to the UAP (Unlicensed assistive personnel)? a. Assess the patient's skin during a bath. b. Reposition the patient using the trapeze. c. Assess the patient's ability to perform range-of-motion exercises. d. Notify the health care provider of any changes.

ANS: B Repositioning a patient can be delegated to unlicensed assistive personnel (UAP); the nurse should provide proper instruction regarding specific positioning techniques, individualized patient concerns, and circumstances that require notifying a nurse. UAP may not perform assessments or evaluations but should notify the nurse about any skin or musculoskeletal issues (not the health care provider).

10. The nurse is teaching a student nurse about restraint use in patients. Which statement by the student nurse indicates a learning need regarding restraints? a. "Having all four side rails up on the bed is considered a restraint." b. "The use of restraints has been shown to decrease fall-related injuries." c. "Death has been associated with the use of restraints." d. "Medications administered to control behavior are considered a chemical restraint."

ANS: B Restraints may be physical or chemical. A physical restraint is a mechanical or physical device, such as material or equipment attached or adjacent to the patient's body, used to restrict movement. Examples of physical restraints are wrist or ankle restraints, a jacket or vest, and side rails. A medication that is administered to a patient to control behavior is a chemical restraint. The use of restraints has been associated with patient injury including death and does not prevent patient falls.

4. A nurse wants to volunteer for a community group providing secondary prevention. What activity would the nurse attend? a. Stroke rehabilitation support group b. Blood pressure screening at the mall c. Bicycle safety class at the elementary school d. Drop by nutrition station at the grocery store

ANS: B Secondary prevention activities are aimed at early diagnosis and prompt intervention. Blood pressure screening events are a good example. Stroke rehabilitation is tertiary prevention. Bicycle safety classes and nutrition education are examples of primary prevention.

10. The patient is admitted to the emergency department of the local hospital from home with reports of chest discomfort and shortness of breath. The patient is placed on oxygen, has labs and blood gases drawn, and is given an electrocardiogram and breathing treatments. Which level of preventive care is this patient receiving? a. Primary prevention b. Secondary prevention c. Tertiary prevention d. Health promotion

ANS: B Secondary prevention focuses on individuals who are experiencing health problems or illnesses and who are at risk for developing complications or worsening conditions. Activities are directed at diagnosis and prompt intervention. Primary prevention precedes disease or dysfunction and is applied to people considered physically and emotionally healthy. Health promotion includes health education programs, immunizations, and physical and nutritional fitness activities for healthy people. Tertiary prevention occurs when a defect or disability is permanent and irreversible. It involves minimizing the effects of long-term disease or disability through interventions directed at preventing complications and deterioration.

16. The nurse is caring for a patient who is having difficulty understanding the written and spoken word. Which type of aphasia will the nurse report to the oncoming shift? a. Expressive b. Receptive c. Global d. Motor

ANS: B Sensory or receptive aphasia is the inability to understand written or spoken language. A patient is able to express words but is unable to understand questions or comments of others. Expressive aphasia, a motor type of aphasia, is the inability to name common objects or express simple ideas in words or writing. Global aphasia is the inability to understand language or communicate orally.

7. The nurse recognizes that a patient is using a portable generator in the house as a power source. What source of poisoning does the nurse appropriately identify? a. Lead b. Carbon monoxide c. Antifreeze d. Pesticide

ANS: B Sources of carbon monoxide include automobiles, stoves, gas ranges, portable generators, lanterns, the burning of charcoal and wood, and heating systems. Lead is found in lead-based paints in toys, buildings, and ceramic dishes; sources of lead include water from lead pipes or pipes soldered with lead, gasoline or soil contaminated by gasoline, and household dust that may contain paint chips or soil. Antifreeze and pesticides are liquids.

6. The nurse is monitoring for Never Events. Which finding indicates the nurse will report a Never Event? a. Lack of blood incompatibility with a blood transfusion b. A surgical sponge is left in the patient's incision c. Pulmonary embolism after lung surgery d. Stage II pressure ulcer

ANS: B The Centers for Medicare and Medicaid Services names select serious reportable events as Never Events (i.e., adverse events that should never occur in a health care setting). A surgical sponge left in a patient's incision is a Never Event. No blood incompatibility reaction is safe practice. Pulmonary embolism is more after certain orthopedic procedures like a total knee and hip replacement. Stage III and IV pressure ulcers are Never Events.

2. A nurse is using the World Health Organization definition of health to provide care. Which area will the nurse focus on while providing care? a. Focusing on helping patients be disease free b. Providing care that involves the whole person c. Assuring that care is strictly personal in nature d. Directing focus only on the pathological state

ANS: B The World Health Organization (WHO) defines health as a ―state of complete physical, mental, and social well-being, not merely the absence of disease or infirmity.‖ Therefore, nurses' attitudes toward health and illness should consider the total person, as well as the environment in which the person lives. All people free of disease are not necessarily healthy. Strictly personal and a focus only on pathological states do not correlate to WHO's definition.

16. A nurse is a case manager for a home health care agency. The nurse often orders supplies for patients seen by the agency. What action by the nurse is best? a. Negotiate for cheaper prices from suppliers. b. Investigate what each patient's insurance will cover. c. Refer the patient to the closest supply source. d. Use the same supplier for all patients' needs.

ANS: B The case manager in home health care must be a well-versed financial steward and understand what each patient's insurance will cover to maximize the patient's benefit. The home health care nurse serves as a case manager (coordinator) of client care, needed services, and needed supplies in the home setting. The nurse must be well versed as a financial resource manager, who needs to be aware of what is or is not covered on the client's insurance plan.

22. Which patient will cause the nurse to select a nursing diagnosis of Impaired physical mobility for a care plan? a. A patient who is completely immobile b. A patient who is not completely immobile c. A patient at risk for single-system involvement d. A patient who is at risk for multisystem problems

ANS: B The diagnosis of Impaired physical mobility applies to the patient who has some limitation but is not completely immobile. The diagnosis of Risk for disuse syndrome applies to the patient who is immobile and at risk for multisystem problems because of inactivity. Beyond these diagnoses, the list of potential diagnoses is extensive because immobility affects multiple body systems.

6. A nurse is interested in epidemiology. What work activity would best fit this role? a. Studying census data to determine common causes of death b. Researching population variables that contribute to disease c. Developing sanitary measures to prevent foodborne illness d. Designing research to determine the connection between pollution and cancer

ANS: B The epidemiologist works to develop programs to prevent the development and spread of disease. Studying census data, researching population variables, and designing studies do not fall in this field.

2. The family of a patient who was in a motor vehicle accident tells the nurse "I'm just not the person I was before the crash." The nurse recognizes this is likely because of the injury to what area of brain? a. Parietal lobes b. Frontal lobes c. Occipital lobes d. Temporal lobes

ANS: B The frontal lobes of the cerebrum are the areas of the brain responsible for voluntary motor function, concentration, communication, decision making, and personality. The parietal lobes are responsible for the sense of touch, distinguishing the shape and texture of objects. The temporal lobes are concerned with the senses of hearing and smell. The occipital lobes process visual information.

5. The patient is reporting moderate incisional pain that was not relieved by the last dose of pain medication. The patient is not due for another dose of medication for another 2 1/2 hours. The nurse repositions the patient, asks what type of music the patient likes, and sets the television to the channel playing that type of music. Which health care model is the nurse using? a. Health belief model b. Holistic health model c. Health promotion model d. Maslow's hierarchy of needs

ANS: B The holistic health model recognizes the natural healing abilities of the body and incorporates complementary and alternative interventions such as music therapy. The health belief model addresses the relationship between a person's beliefs and behaviors. The health promotion model notes that each person has unique personal characteristics and experiences that affect subsequent actions. The basic human needs model believes that the extent to which basic needs are met is a major factor in determining a person's level of health. Maslow's hierarchy of needs is a model that nurses use to understand the interrelationships of basic human needs.

10. The nurse is assessing a patient for possible lead poisoning. Which patient is the nurse most likely assessing? a. A teenager b. A toddler c. A young adult d. An adolescent

ANS: B The incidence of lead poisoning is highest in late infancy and toddlerhood. Children at this stage explore the environment and, because of their increased level of oral activity, put objects in their mouths. The other age-groups are less likely to indiscriminately put objects in their mouth.

33. Which patient will the nurse see first? a. A 56-year-old patient with oxygen with a lighter on the bedside table b. A 56-year-old patient with oxygen using an electric razor for grooming c. A 1-month-old infant looking at a shiny, round battery just out of arm's reach d. A 1-month-old infant with a pacifier that has no string around the baby's neck

ANS: B The nurse will see the patient shaving with an electric razor first as this is an actual problem. Do not use oxygen around electrical equipment or flammable products. A lighter on the bedside table and a shiny, round battery are potential problems, not actual. Plus, it would be hard, almost impossible, for a 1-month-old to actually grab the battery when it is out of arm's reach. A baby should use a pacifier without strings.

8. The nurse notices her 50-year-old patient is holding the lunch menu at arm's length while trying to read the choices. The nurse knows this is an indication of which condition? a. Retinopathy b. Presbyopia c. Cataracts d. Macular degeneration

ANS: B The patient demonstrates presbyopia by holding reading materials at a distance or by being unable to read normal-sized or small print. Retinopathy is damage to the retina and occurs in diabetics. Cataracts are a clouding of the lens. Macular degeneration is a chronic condition that causes loss of vision in the center of your field of vision.

29. The patient applies sequential compression devices after going to the bathroom. The nurse checks the patient's application of the devices and finds that they have been put on upside down. Which nursing diagnosis will the nurse add to the patient's plan of care? a. Risk for falls b. Deficient knowledge c. Risk for suffocation d. Impaired physical mobility

ANS: B The patient has a knowledge need and requires instruction regarding the device and its purpose and procedure. The nurse will intervene by teaching the patient about the sequential compression device and instructing the patient to call for assistance when getting up to go to the bathroom in the future, so that the nurse may assist with removal and proper reapplication. No data support a risk for falls, impaired physical mobility, or suffocation.

12. Upon completing a history, the nurse finds that a patient has risk factors for developing lung disease. How should the nurse interpret this finding? a. A person with the risk factor will get the disease. b. The chances of getting the disease are increased. c. Risk modification will have no effect on disease prevention. d. The disease is guaranteed not to develop if the risk factor is controlled.

ANS: B The presence of risk factors does not mean that a disease will develop, but risk factors increase the chances that the individual will experience a particular disease or dysfunction. Control of risk factors does not guarantee that a disease will not develop. However, risk factor modification can assist patients in adopting activities to promote health and decrease risks of illness.

4. Prior to drug administration the nurse reviews the seven rights, which include right patient, right medication, right time, right dose, right education, right documentation, and what other right? a. Room b. Route c. Physician d. Manufacturer

ANS: B The right route (e.g., oral or intramuscular) is an essential component to verify prior to the administration of any drug. The patient does not need to be in a specific location. There may be a number of physicians caring for a patient who prescribe medications for any given patient. A similar drug may be made by a number of different companies, and checking the manufacturer is not considered one of the seven rights. However, the nurse will want to be aware of a difference, because different companies prepare the same medication in different ways with different inactive ingredients, which can affect patient response.

5. A patient requires repositioning every 2 hours. Which task can the nurse delegate to the nursing assistive personnel? a. Determining the level of comfort b. Changing the patient's position c. Identifying immobility hazards d. Assessing circulation

ANS: B The skill of moving and positioning patients in bed can be delegated to nursing assistive personnel (NAP). The nurse is responsible for assessing the patient's level of comfort and for any hazards of immobility and assessing circulation.

4. An adult patient presents to the emergency department and is treated for hypothermia. What risk factor should the patient be assessed for? a. Tobacco use b. Homelessness c. High carbohydrate diet d. History of chronic respiratory disorder

ANS: B The temperature indicates the patient is experiencing hypothermia. Homeless individuals are more at risk for hypothermia. None of the other options are a known risk factor for hypothermia.

15. A female patient has been overweight for most of her life. She has tried dieting in the past and has lost weight, only to regain it when she stopped dieting. The patient is visiting the weight loss clinic/health club because she has decided to do it. She states that she will join right after the holidays, in 3 months. Which stage is the patient displaying? a. Precontemplation b. Contemplation c. Preparation d. Action

ANS: B This patient is planning to make the change within the next 6 months and is in the contemplation stage. These stages range from no intention to change (precontemplation), to considering a change within the next 6 months (contemplation), to making small changes (preparation), to actively engaging in strategies to change behavior (action), to maintaining a changed behavior (maintenance).

7. The patient has been in bed for several days and needs to be ambulated. Which action will the nurse take first? a. Maintain a narrow base of support. b. Dangle the patient at the bedside. c. Encourage isometric exercises. d. Suggest a high-calcium diet.

ANS: B To prevent injury, nurses implement interventions that reduce or eliminate the effects of orthostatic hypotension. Mobilize the patient as soon as the physical condition allows, even if this only involves dangling at the bedside or moving to a chair. A wide base of support increases balance. Isometric exercises (i.e., activities that involve muscle tension without muscle shortening) have no beneficial effect on preventing orthostatic hypotension, but they improve activity tolerance. A high-calcium diet can help with osteoporosis but can be detrimental in an immobile patient.

7. The student studying community health nursing learns that vulnerable populations can be best assisted by which activity? a. Researching their genetic risk for health problems b. Working with the community to decrease health risks c. Studying vital statistics to determine their causes of death d. Making sure the population maintains immunizations

ANS: B Vulnerable populations have some characteristic that puts them at higher risk for identified health problems. The nurse can best assist vulnerable populations by identifying and working with them to decrease their risks. Researching genetic risks, studying vital statistics, and improving immunizations are all part of the solution, but the overarching priority action is to help the community decrease its risks.

1. A patient who has been in the hospital for several weeks is about to be discharged. The patient is weak from the hospitalization and asks the nurse to explain why this is happening. What is the nurse's best response? a. "Your iron level is low. This is known as anemia." b. "Your immobility in the hospital is known as deconditioning." c. "Your poor appetite is known as malnutrition." d. "Your medications have caused drug induced weakness."

ANS: B When a person is ill and immobile the body becomes weak. This is known as deconditioning. Anemia, malnutrition, and medications may have an adverse effect on the body, but this is not known as deconditioning which is the most likely cause in this patient's situation.

11. The nurse is preparing to lift and reposition a patient. Which action will the nurse take first? a. Position a drawsheet under the patient. b. Assess weight to determine assistance needs. c. Delegate the task to a nursing assistive personnel. d. Attempt to manually lift the patient alone before asking for assistance.

ANS: B When lifting, assess the weight you will lift, and determine the assistance you will need. The nurse has to assess before positioning a drawsheet or delegating the task. Manual lifting is the last resort, and it is used when the task at hand does not involve lifting most or all of the patient's weight; most facilities have a no-lift policy.

27. Which goal is most appropriate for a patient who has had a total hip replacement? a. The patient will ambulate briskly on the treadmill by the time of discharge. b. The patient will walk 100 feet using a walker by the time of discharge. c. The nurse will assist the patient to ambulate in the hall 2 times a day. d. The patient will ambulate by the time of discharge.

ANS: B ―The patient will walk 100 feet using a walker by the time of discharge‖ is individualized, realistic, and measurable. ―Ambulating briskly on a treadmill‖ is not realistic for this patient. The option that focuses on the nurse, not the patient, is not a measurable goal; this is an intervention. ―The patient will ambulate by the time of discharge‖ is not measurable because it does not specify the distance. Even though we can see that the patient will ambulate, this does not quantify how far.

7. The nurse is correctly demonstrating the use of a transfer belt when engaging in which actions? (Select all that apply.) a. The belt is placed around the patient's hips. b. The belt is secure, leaving only enough room for the nurse to grasp the belt. c. The nurse stands on the weaker side. d. The nurse holds the belt on the side of the patient. e. The nurse stands behind the patient while ambulating.

ANS: B, C Transfer belts are used for patients with an unsteady gait or generalized weakness. Canvas transfer or gait belts are applied snugly around the patient's waist, leaving only enough room for the nurse to grasp the belt firmly during ambulation. Some belts may have handles. If the patient has a weaker side, the nurse should stand on that side and hold the gait belt firmly at the back of the patient's waist while ambulating.

2. The nurse is providing education to a cardiac patient who has multiple life stressors that are impacting the patient's health. Which statements by the patient indicate a good understanding of actions that can be taken to reduce stressors? (Select all that apply.) a. "I should change my job." b. "I should plan some downtime." c. "I should meet with a financial counselor." d. "I should talk with my family about my situation." e. "I should make my family go to counseling with me."

ANS: B, C, D In adulthood, life stressors such as financial concerns, work-related demands, and efforts to balance work with family life are common challenges that can take a physical toll on the body. Individuals should plan relaxation periods or vacations. Meeting with financial counselors and talking with family can help to achieve that balance. Changing jobs may be beneficial but could also create more stress and forcing family to go to counseling may also not be a wise choice.

1. The nurse is explaining the National Patient Safety Goals (NPSG) to the student nurse. Which answers indicate that the student has a good understanding of these goals? (Select all that apply.) a. The NPSG's focus on treating chronic infections quickly b. The NPGS's focus on improving staff communication c. The NPGS's focus on using medications safely d. The NPGS's focus on identifying patients correctly

ANS: B, C, D The NPSG focus on specific goals each year. The goals for 2018 included: identify patients correctly, improve staff communication, improve the safety of using medications, reduce harm associated with clinical alarm systems, reduce risk of health care-associated infections and the hospital identifies safety risks inherent in its patient population. Although treating chronic infections quickly is important, it is not an NPSG.

4. The nurse is performing the ―Timed Get Up and Go (TUG)‖ assessment. Which mobility issues will this test measure? (Select all that apply.) a. Pain b. Balance c. Walking d. Moving from sitting to standing positions e. Ability to self-report regarding ability to ambulate

ANS: B, C, D The TUG test measures the progress of balance, sit to stand, and walking. Visual observation of the patient is preferred to reliance on self-reporting. Pain is not an evaluation included in this assessment.

2. The nurse is providing education to the family of a patient being discharged with dementia. Which statement by the family indicates an appropriate level of understanding of dementia? (Select all that apply.) a. "The condition is permanent and has an acute onset." b. "Alzheimer is the most common type of dementia." c. "The condition worsens over time." d. "I should observe for wandering behavior." e. "Agitation can be worse in the evening."

ANS: B, C, D, E Dementia, which is a permanent decline in mental function, has a subtle onset. The most common type of dementia is Alzheimer disease. Dementia is not reversible and worsens over time. Behavioral problems that arise in dementia patients include wandering, agitation, repetitive behaviors, and sundowning, or worsening of agitation and confusion in the evening.

6. The nurse is caring for a diabetic patient who has had a long history of poor glucose control. For what complications is the patient at risk? (Select all that apply.) a. Sudden loss of consciousness b. Diabetic retinopathy c. Stroke d. Peripheral neuropathy e. Memory loss

ANS: B, C, D, E Long-term complications of hyperglycemia may contribute to cognitive and sensory deficits such as memory loss. They also can lead to diabetic retinopathy, peripheral neuropathy, and stroke. Loss of consciousness is usually seen with hypoglycemia in diabetics.

2. The nurse is caring for an older adult who presents to the clinic after a fall. The nurse reviews fall prevention in the home. Which information will the nurse include in the teaching session? (Select all that apply.) a. Water outdoor plants with a nozzle and hose. b. Walk to the mailbox in the summer. c. Encourage yearly eye examinations. d. Use bathtubs without safety strips. e. Keep pathways clutter free.

ANS: B, C, E Walking to the mailbox in summer provides exercise when pathways are not icy and slick. Encourage annual vision and hearing examinations. Pathways that are clutter free reduce fall risk. Using a hose to water plants and using tubs without safety strips are all items the patient should avoid to help in the prevention of falls in the home.

8. The nurse is caring for a patient in restraints. Which essential information will the nurse document in the patient's medical record to provide safe care? (Select all that apply.) a. Family member has left room and gone to lunch. b. Patient is placed in bilateral wrist restraints at 0815. c. Bilateral radial pulses present, 2+, hands warm to touch. d. Straps with quick-release buckle attached to bed side rails. e. Attempts to distract the patient with television are unsuccessful. f. Released from restraints, active range-of-motion exercises completed.

ANS: B, C, E, F Proper documentation, including the behaviors that necessitated the application of restraints, the procedure used in restraining, the condition of the body part restrained (e.g., circulation to hand), and the evaluation of the patient response, is essential. Record nursing interventions, including restraint alternatives tried, in nurses' notes. Record purpose for restraint, type and location of restraint used, time applied and discontinued, times restraint was released, and routine observations (e.g., skin color, pulses, sensation, vital signs, and behavior) in nurses' notes and flow sheets. Straps are not attached to side rails. Comments about the activities of one family member are not necessarily required in nursing documentation of restraints.

5. The nurse knows which findings indicate orthostatic hypotension? (Select all that apply.) a. A decrease in systolic blood pressure by 30 mm Hg b. A decrease in diastolic blood pressure by 10 mm Hg c. An increase in heart rate by 30 beats/min d. An increase in systolic blood pressure by 20 mm Hg e. A decrease in heart rate by 20 beats/min

ANS: B, D A drop in systolic blood pressure of 20 mm Hg, an increase in heart rate of 20 beats/min, or a drop of diastolic blood pressure of 10 mm Hg when a patient stands is classified as orthostatic hypotension.

3. A patient requires restraints after alternatives are not successful. The nurse is reviewing the orders. Which findings indicate to the nurse the order is legal and appropriate for safe care? (Select all that apply.) a. Health care provider orders restraints prn (as needed). b. Health care provider writes the type and location of the restraint. c. Health care provider renews orders for restraints every 24 hours. d. Health care provider performs a face-to-face assessment prior to the order. e. Health care provider specifies the duration and circumstances under which the restraint will be used.

ANS: B, D, E A physician's/health care provider's order is required, based on a face-to-face assessment of the patient. The order must be current, state the type and location of restraint, and specify the duration and circumstances under which it will be used. These orders need to be renewed within a specific time frame according to the policy of the agency. In hospital settings each original restraint order and renewal is limited to 8 hours for adults, 2 hours for ages 9 through 17, and 1 hour for children under age 9. Restraints are not to be ordered prn (as needed).

1. A 65-year-old female patient has been admitted to the medical/surgical unit. The nurse is assessing the patient's risk for falls so that falls prevention can be implemented if necessary. Select all the risk factors that apply from this patient's history and physical. (Select all that apply.) a. Being a woman b. Taking more than six medications c. Having hypertension d. Having cataracts e. Muscle strength 3/5 bilaterally f. Incontinence

ANS: B, D, E, F Adverse effects of medications can contribute to falls. Cataracts impair vision, which is a risk factor for falls. Poor muscle strength is a risk factor for falls. Incontinence of urine or stool increases risk for falls. Men have a higher risk for falls. Hypertension itself does not contribute to falls. Taking medications to treat hypertension that may lead to hypotension and dizziness is a fall risk. Dizziness does contribute to falls.

3. The nurse is educating the patient about the effects of immobility on the body. Which statements by the patient indicate a need for further education? (Select all that apply.) a. "I can become very weak." b. "I will gain weight." c. "I will lose muscle tone." d. "I can get bed sores." e. "I won't have any lung problems."

ANS: B, E Immobility may cause weakness, instability, anorexia, elimination alterations, decreased muscle tone, circulatory stasis, DVTs, pulmonary embolism, and skin breakdown. Knowing the effects of immobility on various body systems allows the nurse to quickly assess a patient's risk and recognize signs of impending complications.

3. The economic stability of individuals or families can determine whether they are willing to seek preventive care or screening examinations. The nurse knows which statements about screening examinations to be true? (Select all that apply.) a. Free or low-cost screening ensures patient screening. b. People may not screen due to fear of testing positive. c. Early screening ensures minimal treatment costs. d. Employment stability is enhanced by early screening. e. Treatment of disorders often means lost wages.

ANS: B, E The economic stability of individuals or families can determine whether they are willing to seek preventive care or screening examinations. Even if screening is free or low cost, the patient or family members may decline because of the potential for testing positive for a disease. Treatment of a disorder often requires time spent away from work, lost wages, and expensive drug therapies and diagnostic tests. The financial impact can be devastating to families or individuals who have a limited or fixed income and fear that employment stability may be compromised.

6. A nurse is preparing to assess a patient for orthostatic hypotension. Which piece of equipment will the nurse obtain to assess for this condition? a. Thermometer b. Elastic stockings c. Blood pressure cuff d. Sequential compression devices

ANS: C A blood pressure cuff is needed. Orthostatic hypotension is a drop of blood pressure greater than 20 mm Hg in systolic pressure or 10 mm Hg in diastolic pressure and symptoms of dizziness, light-headedness, nausea, tachycardia, pallor, or fainting when the patient changes from the supine to standing position. A thermometer is used to assess for fever. Elastic stockings and sequential compression devices are used to prevent thrombus.

33. The nurse is caring for a patient with a spinal cord injury and notices that the patient's hips have a tendency to rotate externally when the patient is supine. Which device will the nurse use to help prevent injury secondary to this rotation? a. Hand rolls b. A trapeze bar c. A trochanter roll d. Hand-wrist splints

ANS: C A trochanter roll prevents external rotation of the hips when the patient is in a supine position. Hand rolls maintain the thumb in slight adduction and in opposition to the fingers. Hand-wrist splints are individually molded for the patient to maintain proper alignment of the thumb and the wrist. The trapeze bar is a triangular device that hangs down from a securely fastened overhead bar that is attached to the bedframe. It allows the patient to pull with the upper extremities to raise the trunk off the bed, to assist in transfer from bed to wheelchair, or to perform upper arm exercises.

15. The nurse is creating a plan of care for a patient diagnosed with glaucoma. Which nursing diagnosis will the nurse include in the care plan to address a safety complication of the sensory deficit? a. Body image disturbance b. Impaired socialization c. Risk for falls d. Fear

ANS: C A visual disturbance poses great risk for injury due to falling from impaired depth perception and inability to see obstacles. Body image disturbance, social isolation, and fear are all valid nursing diagnoses that apply to a patient with vision deficit; however, they do not address the greatest risk for injury.

6. What percentage of hip fractures is the result of falls? a. 50% b. 80% c. 90% d. 100%

ANS: C About 90% of falls end with a hip fracture.

31. The nurse needs to move a patient up in bed using a drawsheet. The nurse has another nurse helping. In which order will the nurses perform the steps, beginning with the first one? 1. Grasp the drawsheet firmly near the patient. 2. Move the patient and drawsheet to the desired position. 3. Position one nurse at each side of the bed. 4. Place the drawsheet under the patient from shoulder to thigh. 5. Place your feet apart with a forward-backward stance. 6. Flex knees and hips and on the count of three shift weight from the front to back leg. a. 1, 4, 5, 6, 3, 2 b. 4, 1, 3, 5, 6, 2 c. 3, 4, 1, 5, 6, 2 d. 5, 6, 3, 1, 4, 2

ANS: C Assisting a patient up in bed with a drawsheet (two or three nurses): (1) Place the patient supine with the head of the bed flat. A nurse stands on each side of the bed. (2) Remove the pillow from under the patient's head and shoulders and place it at the head of the bed. (3) Turn the patient side to side to place the drawsheet under the patient, extending it from shoulders to thighs. (4) Return the patient to the supine position. (5) Fanfold the drawsheet on both sides, with each nurse grasping firmly near the patient. (6) Nurses place their feet apart with a forward-backward stance. Nurses should flex knees and hips. On the count of three, nurses should shift their weight from front to back leg and move the patient and drawsheet to the desired position in the bed.

28. The nurse is working on an orthopedic rehabilitation unit that requires lifting and positioning of patients. Which personal injury will the nurse most likely try to prevent? a. Arm b. Hip c. Back d. Ankle

ANS: C Back injuries are often the direct result of improper lifting and bending. The most common back injury is strain on the lumbar muscle group. While arm, hip, and ankle can occur, they are not as common as back.

2. When teaching a patient with a family history of hypertension about health promotion, the nurse describes blood pressure screening as which type of prevention? a. Illness b. Primary c. Secondary d. Tertiary

ANS: C Blood pressure screening is considered secondary prevention. It is a measure designed to identify individuals in an early state of a disease process so that prompt treatment can be started. Illness prevention is considered primary prevention. Primary prevention measures are those strategies aimed at optimizing health and disease prevention in general and not linked to a single disease entity. Tertiary prevention measures are those that minimize the effects of disease and disability.

14. The nurse is providing discharge instructions to a patient with visual alterations. Which statement by the patient indicates a need for further education? a. "I should make sure the passageways are wide." b. "I should remove all the throw rugs." c. "I should keep the lights dim." d. "I can use a cane to feel for objects in front of me."

ANS: C Bright lighting in hallways and stairways prevents falls by the patient who has limited vision. Furniture is placed to allow wide passageways. Throw rugs, which are a tripping hazard, are removed. If vision is severely limited, use of a cane or walking stick held slightly in front helps the patient feel objects in his/her path.

2. An older patient is talking with the nurse about hip fractures. The patient would like to know the best approach to strengthen the bones. What is the nurse's best response? a. "Walk at least 5 miles every day for exercise." b. "Wear proper fitting shoes to prevent tripping." c. "Talk with your physician about a calcium supplement." d. "Stand up slowly so you don't feel faint."

ANS: C Calcium strengthens the bones. A calcium supplement will help strengthen bones as they may be affected by aging, illness, or trauma. Walking several miles will help strengthen the bones, but the patient should consult with the healthcare provider before any exercise regimen is implemented for the older adult. Wearing proper shoes and standing slowly to prevent dizziness is important but they will not prevent fractures.

21. The nurse correctly recognizes which one of the following illnesses to trigger the broadest range of emotional and behavioral responses? a. Ear infection b. Mild concussion c. Rheumatoid arthritis d. Influenza

ANS: C Chronic, debilitating disease such as rheumatoid arthritis and severe illness can produce a broad range of emotional or behavioral responses in patients and their families. A short-term, self-limited illness that is not life threatening does not evoke emotions or actions that cause fundamental changes in daily lifestyle. More often, illnesses such as the flu, ear infections, and sore throats are viewed as minor irritations or inconveniences. They usually require a short-term adjustment in daily routines, and treatment of symptoms is the priority so that the individual can continue with normal activities. The emotional and behavioral changes associated with non-life-threatening illness are usually minimal, and the individual quickly returns to the previous baseline level of emotional functioning.

19. A nurse is assessing the skin of an immobilized patient. What will the nurse do? a. Assess the skin every 4 hours. b. Limit the amount of fluid intake. c. Use a standardized tool such as the Braden Scale. d. Have special times for inspection so as to not interrupt routine care.

ANS: C Consistently use a standardized tool, such as the Braden Scale. This identifies patients with a high risk for impaired skin integrity. Skin assessment can be as often as every hour. Limiting fluids can lead to dehydration, increasing skin breakdown. Observe the skin often during routine care.

3. The nurse is educating the family of a patient in the intensive care unit about the patient's cognitive status, including the current problem of delirium. Which statement by the family indicates a need for further education? a. "The delirium can be caused by sensory overload." b. "The delirium is reversible." c. "The delirium is a mood disorder." d. "The delirium is a state of confusion."

ANS: C Delirium is a reversible state of acute confusion. It is characterized by a disturbance in consciousness or a change in cognition that develops over 1 to 2 days and is caused by a medical condition. Delirium may occur in intensive care patients as a result of sensory overload. It is not a mood disorder.

8. The nurse is educating the patient about the proper disposal of medications in the home. Which statement by the patient indicates a good understanding of the information? a. "Remove the label from the bottle and throw in the trash." b. "Flush the medication down the disposal." c. "Mix the medications with kitty litter, place the mixture in a jar, and put the jar in the trash." d. "Dissolve the medication in water and pour down the drain."

ANS: C Flushing or pouring the medication down the drain can contaminate the water system. Throwing the medication in the trash poses potential for someone to remove the medication and use it. This can be avoided by mixing it with an undesirable substance like kitty litter or coffee grounds.

6. The nurse recognizes that intentional behaviors to circumvent illness, detect it early, and maintain the best possible level of mental and physiologic function within the boundaries of illness is the definition of which term? a. Health promotion b. Self-actualization c. Health protection d. Self-transcendence

ANS: C Health protection includes intentional behaviors aimed at circumventing illness, detecting it early, and maintaining the best possible level of mental and physiologic function within the boundaries of illness. Health promotion is behavior motivated by the desire to increase well-being and optimize health status. Maslow considered self-actualization the highest level of optimal functioning and involves the integration of cognition, consciousness, and physiologic utility in a single entity. In later years, Maslow described a level above self-actualization called self-transcendence. He refers to self-transcendence as a peak experience, in which analysis of reality or thought changes a person's view of the world and his or her position in the greater structure of life.

1. A nurse is following the goals of Healthy People 2030 to provide care. Which action should the nurse take? a. Allowing people to continue current behaviors to reduce the stress of change. b. Focusing only on health changes that will lead to better local communities. c. Promoting a society in which all people live long, healthy lives. d. Focusing on illness treatment to provide fast recuperation.

ANS: C Healthy People sets objectives to help the United States increase its focus on health promotion and disease prevention (instead of illness care) and encourages cooperation among individuals, communities, and other public, private, and nonprofit organizations to improve health. The current publication, Healthy People 2030, promotes a society in which all people live long, healthy lives. The goals do not include continuing current behaviors to reduce stress, focusing only on health changes for communities, or focusing on fast recuperation.

17. The ER nurse is triaging a patient with suspected poisoning. Who should the nurse anticipate contacting first? a. Family services b. Radiology c. Poison Control Center d. Respiratory

ANS: C If poisoning is suspected, the National Poison Control Center should be contacted immediately. This information will be needed to determine treatment. Respiratory may be needed, and radiology and family services may also be needed, but that will be determined after the treatment plan is determined.

8. A patient has both hearing and visual sensory impairments. Which psychological nursing diagnosis will the nurse add to the care plan? a. Risk for falls b. Self-care deficit c. Impaired socialization d. Impaired physical mobility

ANS: C In focusing on the psychological aspect of care, the nurse is most concerned about social isolation for a patient who may have difficulty communicating owing to visual and hearing impairment. Self-care deficit, impaired physical mobility, and fall risk are physiological risks for the patient.

15. A patient presents to the clinic for illness, and the sick role is legitimized by the provider. The nurse recognizes this as what stage of illness according to Suchman's Model? a. I b. II c. III d. IV

ANS: C In stage III (Medical care contact), professional advice from health care providers is sought by the individual. A professional health care provider identifies and validates the illness and legitimizes the sick role. During stage II (Assumption of the sick role), the person decides that the illness is genuine and that care is necessary. This stage gives an individual permission to act sick and to be excused temporarily from typical social and personal obligations. During stage I (Symptom experience), a clinical manifestation of disease is experienced, and the person acknowledges that something is wrong and seeks a cure. The outcome of stage I is that the person accepts the reality of symptoms and decides to take action in seeking care. During stage IV (Dependent patient role), the person, who is designated as a patient, usually undergoes treatment. During this stage, patients often feel dependent on others and may experience ambivalent or fearful thoughts that cause them to reject treatment, the advice of health care providers, and the illness.

7. The nurse is providing education to the patient about isometric exercises. Which statement by the patient indicates a good understanding of these exercises? a. "An example of this type of exercise is walking." b. "An example of this type of exercise is running." c. "An example of this type of exercise is Kegels." d. "An example of this type of exercise is weight lifting."

ANS: C Isometric exercise requires tension and relaxation of muscles without joint movement. An example is tension and relaxation of pelvic floor muscles (i.e., Kegel exercise). Isotonic exercise involves active movement with constant muscle contraction, such as walking, turning in bed, and self-feeding. Aerobic exercise requires oxygen metabolism to produce energy. Patients may engage in rigorous walking or repeated stair climbing to achieve the positive effects of aerobic exercise. Anaerobic exercise builds power and body mass. Without oxygen to produce energy for activity, anaerobic exercise takes place, such as heavy weight lifting.

3. The nurse is educating the family of a patient on falls risk precautions. Which statement by the family indicates a need for further education? a. "I should keep the wheelchair locked unless using it to move Mom." b. "I should leave the bathroom light on as she does at her home." c. "I should leave her slippers by the wheelchair." d. "I should keep her cell phone close to her bed."

ANS: C Leave lights on or off at night, depending on the patient's cognitive status and personal preference. Keep the wheels of any wheeled device (e.g., bed, wheelchair) in the locked position. Keep patient belongings (e.g., tissues, water, urinals, personal items) within the patient's reach. If the patient is ambulatory, require the use of nonskid footwear (socks or shoes).

28. A nurse is teaching a patient about vision. In which order will the nurse describe the pathway for vision, beginning with the first structure? 1. Lens 2. Pupil 3. Retina 4. Cornea 5. Optic nerve a. 2, 1, 4, 5, 3 b. 1, 2, 4, 3, 5 c. 4, 2, 1, 3, 5 d. 5, 2, 4, 1, 3

ANS: C Light rays enter the convex cornea and begin to converge. An adjustment of light rays occurs as they pass through the pupil and lens. Change in the shape of the lens focuses light on the retina. The sensory retina contains the rods and cones (i.e., photoreceptor cells sensitive to stimulation from light). Photoreceptor cells send electrical potentials by way of the optic nerve to the brain.

3. The nurse is developing a plan of care for a patient with a hip fracture. Which model would the nurse use to prioritize the patient's care? a. The Health Belief Model b. Pender's Health Promotion Model c. Maslow's hierarchy of needs d. The Holistic Health Model

ANS: C Maslow's hierarchy of needs describes the relationships between the basic requirements for survival and the desires that drive personal growth and development. The model is most often presented as a pyramid consisting of five levels. The lowest level is related to physiologic needs, and the uppermost level is associated with self-actualization needs, specifically those related to purpose and identity. The Health Belief Model was developed by psychologists Hochbaum, Rosenstock, and Kegels. It explores how patients' attitudes and beliefs predict health behavior. The Health Promotion Model, developed by Pender and colleagues, defines health as a positive, dynamic state of well-being rather than the absence of disease in the physiologic state. Holistic Health Models in nursing care are based on the philosophy that a synergistic relationship exists between the body and the environment. Holistic care is an approach to applying healing therapies. Holistic models focus on the interrelatedness of body and mind.

14. The nurse needs to consider which approach when caring for patients with chronic illness? a. Help the patient face the reality that he will not get better. b. Emphasize to the patient that the illness is not his fault. c. Focus on improving quality of life through preventive behaviors. d. Acknowledge the limitations placed on the patient by his suffering.

ANS: C Nurses can help patients establish a daily routine of care by educating them about how to manage their care and the symptoms associated with the condition, including emergency or life-threatening situations. Emphasis is on improving quality of life through preventive behaviors. The attitude of being a victim, suffering with, or being afflicted by a chronic illness is viewed by nurses as a counterproductive behavior that needs positive intervention. Nurses can assist patients with strategies that help them cope with their chronic conditions and associated feelings of anger, frustration, and depression. Encouragement and positive support from a professional nurse can help individuals gain control over the alternating periods of health and illness and improve their quality of life.

22. A nurse is providing care to a patient. Which action indicates the nurse is following the National Patient Safety Goals? a. Identifies patient with one identifier before transporting to x-ray department. b. Initiates an intravenous (IV) catheter using clean technique on the first try. c. Uses medication bar coding when administering medications. d. Obtains vital signs to place on a surgical patient's chart.

ANS: C One of the National Patient Safety Goals is to use medicines safely. For example, proper preparation and administration of medications, use of patient and medication bar coding, and ―smart‖ intravenous (IV) pumps reduce medication errors. Identifying patients correctly is a national patient safety goal, and two identifiers are needed, not one. Another goal is to prevent infection; starting an IV should be a sterile technique, not a clean technique. While obtaining vital signs is a component of safe care, it does not meet a national patient safety goal.

6. A nurse is caring for an older-adult patient who was in a motor vehicle accident because the patient thought the stoplight was green. The patient asks the nurse ―Should I stop driving?‖ Which response by the nurse is most therapeutic? a. ―Yes, you should stop driving. As you age, your cognitive function declines, and becoming confused puts everyone else on the road at risk.‖ b. ―Yes, you should ask family members to drive you around from now on. Your reflex skills have declined so much you can't avoid an accident.‖ c. ―No, as you age, you lose the ability to see colors. You need to think about stoplights in a new way. If the top is lit up, it means stop, and if the bottom is lit up, it means go.‖ d. ―No, instead you should see your ophthalmologist and get some glasses to help you see better.‖

ANS: C Part of the normal aging process is reduced ability to see colors. The nurse should teach the patient new ways to adapt to this deficit. This patient's accident was not due to impaired cognitive function or reflexes. Glasses will not assist the patient in color discrimination.

8. The nurse is preparing to assist the patient to walk to the bathroom after medicating the patient with a narcotic for pain management. What possible adverse effect should the nurse be immediately aware? a. Constipation b. Depression c. Dizziness d. Pain relief

ANS: C Potential adverse side effects of narcotics include respiratory depression, hypotension, confusion, sedation, constipation, and dizziness. The nurse should be immediately aware of dizziness during ambulation because of the safety risks. Pain relief is expected. Depression is not an immediate adverse side effect. Constipation will not impact the nurse's ability to safely ambulate the patient.

3. A nurse is caring for a patient diagnosed with presbycusis. Which assessment finding indicates an adaptation to the sensory deficit? a. The patient frequently cleans out eyes with saline washes. b. The patient applies different spices during mealtime to food. c. The patient turns one ear toward the nurse during conversation. d. The patient isolates self from social situations with groups of people.

ANS: C Presbycusis is impaired hearing due to the aging process. Adaptation for a sensory deficit indicates that the patient alters behavior to accommodate for the sensory deficit, such as turning the unaffected ear toward the speaker. Cleaning the eye and applying spices to food would not have an effect for a patient with presbycusis. Avoiding others because of a sensory deficit is maladaptive.

1. The nursing instructor asks the student nurse to identify what Robert Wood Johnson Foundation funded project that focuses on nurses' increased attention to patient safety? a. OSHA (Occupational Safety and Health Agency) b. MSDS (material safety data sheets) c. QSEN (Quality and Safety Education for Nurses) d. ADA (Americans with Disability Act)

ANS: C QSEN, or the Quality and Safety Education for Nurses, was funded by the RWJ to focus on preparing nurses of the future with the knowledge, skills, and attitudes to advance quality and safety on the job. MSDS are material safety data sheets, OSHA is the Occupational Safety and Health Agency, and ADA is the Americans with Disability Act.

1. What response would the nurse give the patient when questioned about the effect of rheumatoid arthritis on the musculoskeletal system? a. Muscle weakness b. Muscle wasting c. Joint inflammation d. Joint spasticity

ANS: C Rheumatoid arthritis and osteoarthritis cause inflammation of joints, resulting in pain and limited joint mobility, not muscle mobility. Genetic disorders such as muscular dystrophy result in muscle weakness and gradual muscle wasting. Spasticity (increased muscle tone) occurs in developmental disorders, such as cerebral palsy, and results in reduced range of motion (ROM) and abnormal movement patterns.

11. A 40-year-old patient presents to her provider for a yearly physical. The provider notes a family history of breast cancer in the patient's mother. The provider schedules the patient for a mammogram. The nurse recognizes this as what level of prevention? a. Tertiary b. Primary c. Secondary d. Holistic

ANS: C Secondary prevention is undertaken in cases of latent (hidden) disease. Although the patient may be asymptomatic, the disease process can be detected by medical tests. Nurses may use screening tests to assess for latent disease in vulnerable populations. Examples of screening tests used as secondary prevention strategies include the purified protein derivative (PPD) skin test for tuberculosis, fecal occult blood test for colorectal cancer, and mammograms for breast cancer. Primary prevention is instituted before disease becomes established by removing the causes or increasing resistance. Examples include the use of seatbelts and airbags in automobiles, helmet use when riding bicycles or motorcycles, and the occupational use of mechanical devices when lifting heavy objects. Tertiary prevention, also known as the treatment or rehabilitation stage of preventive care, is implemented when a condition or illness is permanent and irreversible. The aim of care is to reduce the number and impact of complications and disabilities resulting from a disease or medical condition. Interventions are intended to reduce suffering caused by poor health and assist the patients in adjusting to incurable conditions. Nursing care is focused on rehabilitation efforts in the tertiary stage of prevention. Holistic care is an approach to applying healing therapies. Nurses participate in holistic care through the use of natural healing remedies and complementary interventions. These include the use of art and guided imagery, therapeutic touch, music therapy, relaxation techniques, and reminiscence.

4. The nurse recognizes conversations about safe sexual practices, including the consequences of unprotected sex such as pregnancy and sexually transmitted infections, are important to begin in what patient population? a. Adults b. School-aged children c. Adolescents d. Older adults

ANS: C Sexual curiosity and experimentation occur in the adolescent patient population. Conversations about safe sexual practices, including the consequences of unprotected sex, such as pregnancy and sexually transmitted infections, are important. These conversations are also important for adults and older adults but are handled differently in context with their age-related needs. School-aged children may be too young depending on their age and their environment. The nurse must use judgment on when to have the conversation.

4. The nurse is performing passive range-of-motion exercises on a patient when the patient begins to complain of pain. What is the first thing the nurse should do? a. Notify the health care provider. b. Hyperextend the joint. c. Stop the range of motion. d. Switch to active range of motion.

ANS: C Stop range-of-motion exercises if the patient begins to complain of pain or if resistance to movement is experienced. Never hyperextend or flex a patient's joints beyond the position of comfort. Active range of motion is when the patient moves the joint. Notifying the health care provider would happen later.

5. The nurse is caring for a patient who is complaining of tingling in the hands and fingers. The nurse knows this is a sign of what electrolyte imbalance? a. Hyponatremia b. Hypernatremia c. Hypocalcemia d. Hypercalcemia

ANS: C Tactile disturbances, such as tingling and numbness around the mouth and in the fingers, are signs of hypocalcemia. Mental changes are associated with both hypercalcemia and hypocalcemia. Both hypernatremia and hyponatremia have symptoms of central nervous system disorder.

5. A nurse is orienting to a new job in a home health care agency and is told that most of her patients need tertiary prevention. What activity does the nurse plan to include in the daily routine? a. Household safety checks b. Well-baby checkups c. Antibiotic administration d. Monthly blood pressure assessments

ANS: C Tertiary care is aimed at people who are already experiencing a health alteration, such as those with an infection who need antibiotics. The other options are secondary prevention.

11. A patient is admitted to a rehabilitation facility following a stroke. The patient has right-sided paralysis and is unable to speak. The patient will be receiving physical therapy and speech therapy. Which level of preventive care is the patient receiving? a. Primary prevention b. Secondary prevention c. Tertiary prevention d. Health promotion

ANS: C Tertiary prevention occurs when a defect or disability is permanent and irreversible. It involves minimizing the effects of long-term disease or disability through interventions directed at preventing complications and deterioration. Secondary prevention focuses on individuals who are experiencing health problems or illnesses and who are at risk for developing complications or worsening conditions. Activities are directed at diagnosis and prompt intervention. Primary prevention precedes disease or dysfunction and is applied to people considered physically and emotionally healthy. Health promotion includes health education programs, immunizations, and physical and nutritional fitness activities.

5. The home care nurse is trying to determine the necessary services for a 65-year-old patient who was admitted to the home care service after left knee replacement. Which tool is the best for the nurse to utilize? a. Minimum Data Set (MDS) b. Functional Status Scale (FSS) c. 24-Hour Functional Ability Questionnaire (24hFAQ) d. The Edmonton Functional Assessment Tool

ANS: C The 24hFAQ assesses the postoperative patient in the home setting. The MDS is for nursing home patients. The FSS is for children. The Edmonton is for cancer patients.

22. Which statement by the nurse correctly identifies the UAP role in patient restraint use? a. "The UAP can perform initial assessment." b. "The UAP can apply a restraint." c. "The UAP can assist with applying and monitoring of a physical restraint." d. "The UAP can contact the health care provider and request an order for restraints."

ANS: C The UAP cannot perform the initial assessment, and most facilities require that a registered nurse or licensed practical nurse. Applying a restraint. The health care provider should be contacted by the nurse, not the UAP. The UAP can assist with applying the restraint and can perform monitoring checks under the direction of a Registered Nursing.

1. The nurse knows the World Health Organization defines health in which of the following terms? a. The absence of disease b. The lack of infirmity c. Complete well-being d. Being independent of fiscal responsibility

ANS: C The World Health Organization offers a definition for health: "a state of complete physical, mental, and social well-being and not merely the absence of disease or infirmity." Nurses are responsible for helping patients reach their optimal levels of physiologic and mental health, but they also must provide health care in a system that requires cost containment and fiscal responsibility.

24. The nurse is caring for a patient who is a well-known surgeon at the hospital. The nurse notices the patient becoming more agitated and withdrawn with each group of surgeon visitors. The nurse and patient agree to place a ―Do not disturb‖ sign on the door. A few hours later, the nurse notices a surgeon who is not involved in the patient's care attempting to enter the room. Which response by the nurse is most appropriate? a. Call for security to remove the surgeon. b. Allow the surgeon to enter. c. Firmly explain that the patient does not wish to have visitors at this time. d. Scold the surgeon for not obeying the sign and respecting the patient's wishes.

ANS: C The nurse acts as an advocate for the patient (who is experiencing sensory overload and would benefit from a quiet environment) by firmly and politely asking the surgeon to leave regardless of position in the hospital. A creative solution to decrease excessive environmental stimuli that prevents restful, healing sleep is to institute ―quiet time‖ in ICUs. Data collected from one hospital that implemented 1 hour of quiet time daily found decreased staff and unit noise and improved patient satisfaction. The nurse should not allow anyone to enter unless the patient approves it. Security is not a necessary measure at this time. The nurse should handle the situation with professionalism when addressing the surgeon; scolding the visitor is not appropriate.

25. Which behavior indicates the nurse is using a team approach when caring for a patient who is experiencing alterations in mobility? a. Delegates assessment of lung sounds to nursing assistive personnel. b. Becomes solely responsible for modifying activities of daily living. c. Consults physical therapy for strengthening exercises in the extremities. d. Involves respiratory therapy for altered breathing from severe anxiety levels.

ANS: C The nurse should collaborate with other health care team members such as physical or occupational therapists when considering mobility needs. For example, physical therapists are a resource for planning ROM or strengthening exercises. Nurses often delegate some interventions to nursing assistive personnel, but assessment of lung sounds is the nurse's responsibility. Nursing assistive personnel may turn and position patients, apply elastic stockings, help patients use the incentive spirometer, etc. Occupational therapists are a resource for planning activities of daily living that patients need to modify or relearn. A mental health advanced practice nurse or psychologist should be used for severe anxiety.

1. A nurse is caring for a patient with a stroke that has altered her ability to see. The nurse knows which area of the brain was likely impacted by the stroke that is responsible for visual function? a. Parietal lobes b. Frontal lobes c. Occipital lobes d. Temporal lobes

ANS: C The occipital lobes process visual information. The frontal lobes of the cerebrum are the areas of the brain responsible for voluntary motor function, concentration, communication, decision making, and personality. The parietal lobes are responsible for the sense of touch, distinguishing the shape and texture of objects. The temporal lobes are concerned with the senses of hearing and smell.

6. The nurse identifies which goal to be appropriate for the patient who is postoperative day one from abdominal surgery and on bed rest with the nursing diagnosis impaired skin integrity? a. Patient will ambulate twice a day. b. Patient will eat 50% of meals. c. Patient will have no further skin breakdown. d. Patient will interact with others.

ANS: C The patient already has a wound, so the goal is focused on no further skin breakdown as a result of the bed rest and immobility. Although nutrition is important to wound healing, it is not the focus of this Nursing diagnosis. Ambulating and interacting with others are not goals for this diagnosis.

10. The nurse identifies which goal to be most appropriate for the Nursing diagnosis of acute confusion? a. The patient will use the call light before getting out of bed within 48 hours. b. The patient will use a calendar to remember the date within 48 hours. c. The patient will respond appropriately to questions about place within 48 hours. d. The patient will remain within the unit while in long-term care.

ANS: C The patient has acute confusion and therefore an appropriate early goal as the confusion resolves is to remember where they are. Remembering to use a call light would be appropriate for risk for falls. Using a calendar is appropriate for impaired memory and remaining in the unit is appropriate for chronic confusion.

9. A community was devastated by a tornado several months ago. What nursing diagnosis would be most appropriate for the nurse to consider? a. Social isolation b. Deficient community resources c. Ineffective community coping d. Deficient community health

ANS: C This diagnosis considers those in a community who may be feeling helpless, hopeless, or frustrated because of an extraordinary event. Financial and physical resources may not be available for rebuilding. Social isolation refers to unacceptable social behavior. Deficient community resources is not an approved diagnosis. Deficient community health may become a problem if sanitary conditions lead to an outbreak of disease.

16. Which statement by the patient indicates to the nurse a teaching need regarding safety in the home? a. "I will put a night-light in every room." b. "I will not use an extension cord to plug in multiple items." c. "I will wash my throw rugs in the bathroom regularly." d. "I will keep all cleaning supplies out of reach of children."

ANS: C Throw rugs present a fall or tripping hazard. Night-lights help light halls to prevent falls, extension cords can present a trip hazard, and cleaning supplies can contain poisonous materials.

11. Which assessment question should the nurse ask to best understand how visual alterations are affecting the patient's self-care ability? a. ―Have you stopped reading books or switched to books on audiotape?‖ b. ―What do you do to protect yourself from injury at work?‖ c. ―Are you able to prepare a meal or write a check?‖ d. ―How does your vision impairment make you feel?‖

ANS: C To best understand how vision is affecting self-care ability, the nurse wants to target questions to encompass what self-care tasks the patient has difficulty doing, such as preparing meals and writing checks. Switching to books on audiotape gives the nurse an idea of the severity of the deficit but not its impact on activities of daily living. Assessing whether the patient is taking measures for protection is important, but this does not address self-care activities. Emotional assessment of a patient is also important but does not properly address the goal of determining the effect of visual alterations on self-care ability.

13. The nurse is delegating care to an unlicensed assistive personnel (UAP) to a patient who has sensory overload. Which statement by the UAP indicates a need for further orientation? a. "I should keep the noise levels low." b. "I should schedule all the care together." c. "I should keep the room well lit." d. "I should allow the family to visit."

ANS: C To prevent or alleviate overload, the nurse reduces sensory stimuli, dimming unnecessary lights and turning down the sound on alarms if possible. Nursing care is planned so that the patient is not constantly disturbed. Visitation by family provides reality orientation and a soothing, recognizable presence for some patients experiencing overload.

5. Which nursing action indicates that a nurse is more likely to incur a medication error during medication administration? a. Checks the original medication order on the patient's chart b. Asks the patient to state his/her name and date of birth c. Does not scan the barcode of the patient prior to administering the medication d. Does not provide the patient with a glass of water

ANS: C Use of barcode scanning of both the medication and the patient's hospital band is critical to maintaining safe practice during medication administration. The nurse by not scanning the barcode is not maintaining the required elements and as a result is more likely to incur a medication error. Checking the original order and asking the patient to provide identification are required elements. Not providing a glass of water to the patient is not related to a medication error but does not represent best practice unless the patient is NPO except meds which would require sips of water.

12. The nurse is educating the family to care for a patient at home with cognitive alterations. Which statement by the family indicates a need for further education? a. "I should keep the home free of scissors." b. "I should minimize the number of visitors." c. "I should use push-button door locks." d. "24-hour supervision may become necessary."

ANS: C Use of door locks that require a key may be necessary if the patient wanders. Keep the environment free of hazards such as sharp objects and minimize distractions. If the patient is not safe to be left alone, 24-hour supervision may be necessary.

1. A nurse is administering a vaccine to a child who is visually impaired. After the needle enters the arm, the child says, ―Ow, that was sharp!‖ What conclusion about the child will the nurse come to when interpreting the child's response? a. Sensations are intact. b. Reception is intact. c. Perception is intact. d. Reaction is intact.

ANS: C When a person becomes conscious of a stimulus and receives the information, perception takes place. Perception includes integration and interpretation of stimuli based on the person's experiences. Sensation is a general term that refers to awareness of sensory stimuli through the body's sense mechanisms. Reception begins with stimulation of a nerve cell called a receptor, which is usually for only one type of stimulus such as light, touch, taste, or sound. Reaction is how a person responds to a perceived stimulus.

13. The nurse is caring for a patient who has been trying to quit smoking. The patient has been smoke free for 2 weeks but had two cigarettes last night and at least two this morning. What should the nurse anticipate? a. The patient does not want to and will never quit smoking. b. The patient must pick up the attempt right where the patient left off. c. The patient will return to the contemplation or precontemplation phase. d. The patient will need to adopt a new lifestyle for change to be effective.

ANS: C When relapse occurs, the person will return to the contemplation or precontemplation stage before attempting the change again. The patient cannot pick up the attempt where left off. It is believed that change involves movement through a series of stages (precontemplation, contemplation, preparation, action, and maintenance). Anticipating that the patient does not want to and will never quit is premature. While the patient will need to adopt a new lifestyle for change to be effective, it does not correlate to this scenario since the patient relapsed.

16. The nurse is admitting an older adult to the surgical unit. What intervention is necessary when determining the safe use of side rails for this patient? a. Explain to the patient the need to call for assistance when side rails are up. b. Discuss whether the patient is accepting of having the side rails up. c. Assess the patient's ability to effectively follow instructions. d. Always keeping the bed in its lowest position to the floor.

ANS: C When used correctly, side rails help to increase a patient's mobility and/or stability when repositioning or moving in bed or moving from bed to chair. Although side rails are the most commonly used physical restraint, they increase the risk of falls when patients attempt to get out of bed or crawl over a rail. Side rails also can lead to patients becoming caught, trapped, entangled, or strangled, especially in the frail, elderly or confused (FDA, 2017). Therefore, an assessment of a patient's mobility and responsiveness to instructions help determine if using a side rail is safe. Providing patient education, discussing the use of side rails, and proper bed positioning are all appropriate interventions but none are as directly associated with patient safety regarding the use of side rails is the assessment of cognitive understanding.

8. The nurse is correctly assisting the patient in using a cane when the patient demonstrates which activities? (Select all that apply.) a. The top of the cane is level with the patient's bent elbow. b. The patient holds the cane on his/her weaker side. c. The patient moves the cane forward first. d. The patient's arm is comfortably bent when walking. e. The patient moves the strong leg forward first.

ANS: C, D The top of the cane should be level with the hip joint, and the patient's arm should be comfortably bent when the patient is walking. The patient should hold the cane on his/her stronger side and move the cane forward first, followed by the weaker leg and then the stronger leg. This ensures that another point of support is always on the ground when the weaker leg is bearing weight and gives the patient a wide base of support. A patient using a cane should be encouraged to stand up straight and look forward. Leaning to one side or looking down can jeopardize safety and cause poor posture.

10. The nurse is preparing discharge instructions for a patient who has tactile alterations in his legs. Which instructions would be included? (Select all that apply.) a. Verify bath water temperature is approximately 39.5 °C. b. Do not use hot or cold therapy on any extremity. c. Use sturdy shoes when walking outside or on hard surfaces. d. Report any changes in skin color on your legs to your health care provider. e. Set your water heater so that scalding is not possible.

ANS: C, D, E Bath water temperature should be approximately 37.8 °C (100 °F), so 39.5 (103.1 °F) is too hot. Hot and cold therapy should not be used on the affected extremities, although it can be used on other areas of the body. Sturdy shoes can prevent foot injuries when there is decreased sensation in the lower extremities. Any decrease in sensation, change in the color of the skin, or wounds are reported to the health care provider. Water heaters are set so that scalding is not possible.

1. Which areas should the nurse assess to determine the effects of external variables on a patient's illness? (Select all that apply.) a. Patient's perception of the illness b. Patient's coping skills c. Socioeconomic status d. Cultural background e. Social support

ANS: C, D, E External variables influencing a patient's illness behavior include the visibility of symptoms, social group, cultural background, economic variables, accessibility of the health care system, and social support. Internal variables include the patient's perceptions of symptoms and the nature of the illness, as well as the patient's coping skills and locus of control.

8. The nurse is caring for a patient with expressive aphasia. Which interventions will assist the nurse in communicating with the patient? (Select all that apply.) a. Use simple phrases. b. Speak loudly. c. Use yes/no questions. d. Use a picture board. e. Be patient and unrushed.

ANS: C, D, E If a patient has expressive aphasia, he or she understands language but is unable to answer questions, name common objects, or state simple ideas. The patient can answer yes/no questions by shaking the head. The patient might be able to point to pictures to express needs. For any type of aphasia, being patient and not rushing will make communication less stressful.

3. The nurse is caring for a patient who suffered a stroke on the right side of the brain. The nurse is careful to implement what safety measures? (Select all that apply.) a. Puts a picture board in the room to communicate with the patient. b. Places the call light on the patient's left side. c. Leaves a light on in the bathroom at night for good visibility. d. Places the call light on the patient's right side. e. Makes sure there are no trip hazards in the patient's room.

ANS: C, D, E If the damage is on the right side of the brain, there is loss of sensation and motor function in the extremities on the left side of the body and visual-spatial problems occur. Therefore, placing the call light on the side where the patient is likely to be strong is important. The patient will not necessarily have communication problems but might have visual problems, so the bathroom light is helpful. If the damage is on the left side of the brain, there is loss of sensation and motor function in the extremities on the right side of the body and problems with speech occur.

1. The student nurse learns the ANA's Scope and Standards of Practice for public health nursing include components? (Select all that apply.) a. Team membership b. Developing research c. Ethical behavior d. Responsible resource use e. Advocacy

ANS: C, D, E The ANA's Scope and Standards of Practice for public health nursing requires participation in research, responsible resource utilization, ethical behavior, leadership, and advocacy like the standards of practice for all nurses. Team membership and developing one's own research are not included.

5. The community health nurse knows that which are standards of professional performance for home care nurses according to the ANA? (Select all that apply.) a. Collegiality b. Performance appraisal c. Ethical behavior d. Outcome identification e. Resource utilization

ANS: C, E The ANA's Public Health Nursing: Scope and Standards of Practice (2013) requires participation in research, responsible resource utilization, ethical behavior, leadership, and advocacy similar to the standards of practice for all nurses.

1. A nurse manager is reviewing interrelated concepts to professional nursing. Which concepts should the nurse manager consider when addressing concerns about the quality of health promotion? (Select all that apply.) a. Culture b. Development c. Evidence d. Nutrition e. Health policy

ANS: C, E The interrelated concepts to professional nursing include evidence, healthcare economics, health policy, and patient education. Culture is a patient attribute concept. Development is a patient attribute concept. Nutrition is a health and illness concept.

9. A nurse is caring for an older, immobile patient whose condition requires a supine position. Which metabolic alteration will the nurse monitor for in this patient? a. Increased appetite b. Increased diarrhea c. Increased metabolic rate d. Increased pulse rate

ANS: D Lying down increases cardiac workload and results in an increased pulse rate. In older adults, the heart rate often does not tolerate the added workload, and a form of cardiac failure may develop. Immobility disrupts normal metabolic functioning: decreasing the metabolic rate, altering the metabolism of carbohydrates, fats, and proteins; causing fluid, electrolyte, and calcium imbalances; and causing gastrointestinal disturbances such as decreased appetite and slowing of peristalsis, leading to constipation.

20. A patient is admitted and is placed on fall precautions. The nurse teaches the patient and family about fall precautions. Which action will the nurse take in accordance with hospital policy? a. Check on the patient once a shift. b. Encourage visitors in the early evening. c. Place all four side rails in the ―up‖ position. d. Keep the patient on fall risk until discharge.

ANS: D A fall-reduction program includes a fall risk assessment of every patient, conducted on admission and routinely (see hospital policy) until a patient's discharge. The timing of visitors would not affect falls. All four side rails are considered a restraint and can contribute to falling. Individuals on high risk for fall alerts should be checked frequently, at least every hour.

1. A sentinel event refers to which situation? a. An event that could have harmed a patient, but serious harm didn't occur because of chance b. An event that harms a patient as a result of underlying disease or condition c. An event that harms a patient by omission or commission, not an underlying disease or condition d. An event that signals the need for immediate investigation and response

ANS: D A sentinel event is an unexpected occurrence involving death or serious physical or psychological injury or the risk thereof called sentinel, because it signals the need for immediate investigation and response. A near-miss refers to an error or commission or omission that could have harmed the patient, but serious harm did not occur as a result of chance. Harm that relates to an underlying disease or condition provides the rationale for the close monitoring and supervision provided in a healthcare setting. An adverse event is one that results in unintended harm because of the commission or omission of an act.

13. The patient has a nursing diagnosis of Risk for Falls. The nurse identifies which goal to be most important? a. Patient will ambulate twice a day. b. Patient will have no symptoms of infection. c. Patient will perform activities of daily living. d. Patient will have no injuries during hospital stay.

ANS: D All the goals except lack of infection are appropriate for a patient with a Risk for Falls diagnosis; however, the most important goal is for the patient to have no injuries during the hospitalization.

16. When considering factors influencing health and the impact of illness, specifically age, the nurse would correctly identify which patient as having the greatest risk? a. 10-year-old girl b. 23-year-old woman c. 47-year-old man d. 85-year-old woman

ANS: D Assessment of the patient begins with risk factors that take into account the person's age and the associated level of immune system function. The very young, especially neonates and infants born prematurely, are more susceptible to infections because of the immaturity of their immune systems. Likewise, older adults have decreased immune system function because of the aging process. Older patients are at risk for opportunistic infections resulting from harmless organisms that become pathogenic and illness from the spread of community-acquired disease. Complications from comorbidities of chronic disease may also increase suffering in the aged population.

12. The patient asks the nurse to explain collaborative health care partnerships. The nurse gives a correct description when making which statement regarding collaborative care? a. Does not require participation of the patient. b. Is individual and cannot be mandated or legislated. c. Education needs are delegated to assistive personnel. d. Is designed to provide care to the patient as a whole.

ANS: D Collaborative health care partnerships are designed to deliver well-balanced care to the patient as a whole, rather than rendering fragmented care involving a single element of a disease process. Prevention is not solely the responsibility of the nurse; it involves active participation by the individual and the combined services of practitioners in a spectrum of health care disciplines as varied as nutrition, physical therapy, exercise physiology, and pharmacy. Collaborative preventive care can be mandated in the form of health care legislation, with rates for reimbursement of practitioners determined by the individual provider's ability to collaborate and develop innovative methods for delivering high-quality, cost-effective health care services. The role of the professional nurse is to collaborate and communicate health education to the patient and family, care provider, or surrogate. Patient education responsibilities are not delegated to assistive personnel or other members of the health care team and are considered a cornerstone of nursing care.

20. The nurse is caring for a patient in acute respiratory distress. The patient has multiple monitoring systems on that constantly beep and make noise. The patient is becoming agitated and frustrated over the inability to sleep. Which action by the nurse is most appropriate for this patient? a. Administer an opioid medication to help the patient sleep. b. Keep the door open during the night. c. Open the shades at night. d. Provide the patient with earplugs.

ANS: D Control of excessive stimuli becomes an important part of a patient's care; earplugs provide relief. Quiet time means dimming the lights throughout the unit, closing the shades, and shutting the doors. Allow patients to shut their room door to decrease noise. Opioid medications (for pain relief) should not be the first option; however, antianxiety medications and sleep aids may be considered.

14. The nurse has delegated to the UAP to assist a patient with ambulating in the hallway with a cane. Which statement by the UAP indicates a need for further education? a. "I should report any complaints of soreness to the nurse." b. "I should watch for indications that the patient has difficulties using the cane." c. "I should let the nurse or PT know if the cane doesn't seem to fit correctly." d. "I should teach the patient how to walk with the cane."

ANS: D Educating patients on how to walk with assistive devices may not be delegated to unlicensed assistive personnel (UAP). UAP should report any of the following: noticeable incorrect usage or fit of assistive devices, complaints of soreness or weakness, difficulties involving balance or strength, or difficulties in performing the procedure or other concerns verbalized by the patient.

21. The nurse is caring for a patient diagnosed with expressive aphasia from a traumatic brain injury. Which goal will the nurse include in the plan of care? a. Patient will carry a pen and a pad of paper around for communication. b. Patient will recover full use of speech vocabulary in 1 day. c. Patient will thicken drinks to prevent aspiration. d. Patient will communicate nonverbally.

ANS: D Expressive aphasia, a motor type of aphasia, is the inability to name common objects or express simple ideas in words or writing. To adapt to expressive aphasia, the nurse and the patient need to work on ways to communicate nonverbally through means such as pointing and gestures. Goals and outcomes need to be realistic and measurable; recovery in 1 day is not realistic. A patient who has expressive aphasia may not be able to speak or write words with a pen and paper. Thickening drinks prevents aspiration risk and is not included in a plan of care for this patient.

2. A nurse is providing range of motion to the shoulder and must perform external rotation. Which action will the nurse take? a. Moves patient's arm in a full circle. b. Moves patient's arm cross the body as far as possible. c. Moves patient's arm behind body, keeping elbow straight. d. Moves patient's arm until thumb is upward and lateral to head with elbow flexed.

ANS: D External rotation: With elbow flexed, move arm until thumb is upward and lateral to head. Circumduction: Move arm in full circle (Circumduction is combination of all movements of ball-and-socket joint.) Adduction: Lower arm sideways and across body as far as possible. Hyperextension: Move arm behind body, keeping elbow straight.

8. The nurse is working on a committee to evaluate the need for increasing the levels of fluoride in the drinking water of the community. Which concept is the nurse fostering? a. Illness prevention b. Wellness education c. Active health promotion d. Passive health promotion

ANS: D Fluoridation of municipal drinking water and fortification of homogenized milk with vitamin D are examples of passive health promotion strategies. With active strategies of health promotion, individuals are motivated to adopt specific health programs such as weight reduction and smoking cessation programs. Illness prevention activities such as immunization programs protect patients from actual or potential threats to health. Wellness education teaches people how to care for themselves in a healthy way.

1. The nurse is assessing a patient's functional ability. Which patient best demonstrates the definition of functional ability? a. Considers self as a healthy individual; uses cane for stability b. College educated; travels frequently; can balance a checkbook c. Works out daily, reads well, cooks, and cleans house on the weekends d. Healthy individual, volunteers at church, works part time, takes care of family and house

ANS: D Functional ability refers to the individual's ability to perform the normal daily activities required to meet basic needs; fulfill usual roles in the family, workplace, and community; and maintain health and well-being. The other options are good; however, healthy individual, church volunteer, part time worker, and the patient who takes care of the family and house fully meets the criteria for functional ability.

41. The nurse is admitting a patient who has been diagnosed as having had a stroke. The health care provider writes orders for ―ROM as needed.‖ What should the nurse do next? a. Restrict patient's mobility as much as possible. b. Realize the patient is unable to move extremities. c. Move all the patient's extremities. d. Further assess the patient.

ANS: D Further assessment of the patient is needed to determine what the patient is able to perform. Some patients are able to move some joints actively, whereas the nurse passively moves others. With a weak patient, the nurse may have to support an extremity while the patient performs the movement. In general, exercises need to be as active as health and mobility allow.

13. The nurse is observing the way a patient walks. Which aspect is the nurse assessing? a. Activity tolerance b. Body alignment c. Range of motion d. Gait

ANS: D Gait describes a particular manner or style of walking. Activity tolerance is the type and amount of exercise or work that a person is able to perform. Body alignment refers to the position of the joints, tendons, ligaments, and muscles while standing, sitting, and lying. Range of motion is the maximum amount of movement available at a joint in one of the three planes of the body: sagittal, frontal, or transverse.

4. A nurse is performing passive range of motion (ROM) and splinting on an at-risk patient. The absence of which finding will indicate goal achievement for the nurse's action? a. Atelectasis b. Renal calculi c. Pressure ulcers d. Joint contractures

ANS: D Goal achievement for passive ROM is prevention of joint contractures. Contractures develop in joints not moved periodically through their full ROM. ROM exercises reduce the risk of contractures. Researchers noted that prompt use of splinting with prescribed ROM exercises reduced contractures and improved active range of joint motion in affected lower extremities. Deep breathing and coughing and using an incentive spirometer will help prevent atelectasis. Adequate hydration helps prevent renal calculi and urinary tract infections. Interventions aimed at prevention of pressure ulcers include positioning, skin care, and the use of therapeutic devices to relieve pressure.

3. Mobility for the patient changes throughout the lifespan. What is the term that best describes this process? a. Aging and illness b. Illness and disease c. Health and wellness d. Growth and development

ANS: D Growth and development happens from infancy to death. Muscular changes are always happening, and these changes affect the individual and his or her performance in life. Aging, illness, health, and wellness do have an effect on a person, but they don't always affect mobility.

7. The nurse is assessing the patient's ability to hear and knows which is the correct procedure for the doing this? a. The nurse whispers to the patient while standing on each side of the patient. b. The nurse speaks in a normal voice while standing on each side of the patient. c. The nurse speaks in a normal voice while standing directly in front of the patient. d. The nurse speaks in a normal voice while standing slightly behind the patient.

ANS: D Hearing ability can be determined by observing the patient's conversation and responses and by talking with the patient in a normal conversational tone while standing slightly behind the patient. If the patient does not respond appropriately, a hearing impairment may exist. Standing in front of the patient allows the patient to read your lips and will not detect a hearing loss. A whispered voice will also give a false reading.

11. The nurse displays an understanding of high-risk populations for MRSA when identifying which group as the lowest risk? a. Prison inmates b. College dorm residents c. Team athletes d. Food service workers

ANS: D High-risk populations for MRSA include those living in close quarters or those who have frequent skin-to-skin contact, including prison inmates, college dorm residents, and team athletes. Food service workers work together but do not generally live in close quarters or have skin-to-skin contact frequently.

2. The nurse knows changes in which body system affect overall mobility increasing the propensity of falling? a. Neurologic b. Hepatic c. Cardiopulmonary d. Musculoskeletal

ANS: D Impairments in the musculoskeletal system can impact mobility through restrictions of range of motion and strength, increasing the chances of falling. Changes to the neurologic system can impair cognitive functioning, changes to the hepatic system can affect mental status, and changes to the cardiopulmonary system can affect activity tolerance.

9. To promote a culture of safety, the nurse manager preparing the staff schedule considers the anticipated census in planning the number and experience of staff on any given shift. Which is the human factor primarily addressed with this consideration? a. Available supplies b. Interdisciplinary communication c. Interruptions in work d. Workload fluctuations

ANS: D Including an adequate number of staff members with experience caring for anticipated patients is a strategy to manage the workload and potential fluctuations. A safety culture requires organizational leadership (e.g., the nurse manager) that gives attention to human factors such as managing workload fluctuations. This strategy also applies principles of crew resource management in that it addresses workload distribution. Lack of supplies can create a challenge for safe care but could not be addressed with the schedule. Concerns with communication and coordination across disciplines, including power gradients, and excessive professional courtesy can create hazards but would not be the best answer. Strategies to minimize interruptions in work are essential but would not be the best answer in this situation.

3. What statement by the nurse demonstrates an understanding of food safety to be provided for a patient living alone? a. ―It's acceptable to eat unwashed fruits and vegetables if they are organically grown.‖ b. ―It's best to allow cooked foods to thoroughly cool off before putting them into the refrigerator.‖ c. ―You can use the same cutting board for meats and for vegetables if it is washed between uses.‖ d. ―Your perishable left-over food should be stored in a refrigerator at below 40F.‖

ANS: D Keep a refrigerator below 40F to assure proper storage of perishable foods. Keep raw meat, poultry, seafood, and their juices away from other foods. Use separate cutting boards. Rinse fruits and vegetables thoroughly. Refrigerate leftovers promptly.

10. A nurse is preparing a care plan for a patient who is immobile. Which psychosocial aspect will the nurse assess for? a. Loss of bone mass b. Loss of strength c. Loss of weight d. Loss of hope

ANS: D Loss of hope is a psychosocial aspect. Patients with restricted mobility may have some depression. Depression is an affective disorder characterized by exaggerated feelings of sadness, melancholy, dejection, worthlessness, emptiness, and hopelessness out of proportion to reality. All the rest are physiological aspects: bone mass, strength, and weight.

9. A nurse reviews the history of a newly admitted patient. Which finding will alert the nurse that the patient is at risk for falls? a. 55 years old b. 20/20 vision c. Urinary continence d. Orthostatic hypotension

ANS: D Numerous factors increase the risk of falls, including a history of falling, being age 65 or over, reduced vision, orthostatic hypotension, lower extremity weakness, gait and balance problems, urinary incontinence, improper use of walking aids, and the effects of various medications (e.g., anticonvulsants, hypnotics, sedatives, certain analgesics).

3. A nurse is providing passive range of motion (ROM) for a patient with impaired mobility. Which technique will the nurse use for each movement? a. Each movement is repeated 5 times by the patient. b. Each movement is performed until the patient reports pain. c. Each movement is completed quickly and smoothly by the nurse. d. Each movement is moved just to the point of resistance by the nurse.

ANS: D Passive ROM exercises are performed by the nurse. Carry out movements slowly and smoothly, just to the point of resistance; ROM should not cause pain. Never force a joint beyond its capacity. Each movement needs to be repeated 5 times during the session. The patient moves all joints through ROM unassisted in active ROM.

4. The nurse is talking to the unlicensed assistive personnel about moving a patient in bed. The nurse knows the unlicensed assistive personnel understands the concept of mobility and proper moving techniques when making which statement? a. "Patients must have a trapeze over the bed to move properly." b. "Patients should move themselves in bed to prevent immobility." c. "Patients should always have a two-person assist to move in bed." d. "Patients must be moved correctly in bed to prevent shearing."

ANS: D Patients must be moved properly in bed to prevent shearing of the skin. Having a trapeze over the bed is only functional if the patient can assist in the moving process. A two-person assist is good, but the patient still needs to be moved properly. A patient may move himself or herself if he or she is able; but shearing may still occur.

10. The nurse knows that use of seatbelts and airbags in automobiles is an example of which term? a. Secondary prevention b. Tertiary prevention c. Holistic care d. Primary prevention

ANS: D Primary prevention is instituted before disease becomes established by removing the causes or increasing resistance. Examples include the use of seatbelts and airbags in automobiles, helmet use when riding bicycles or motorcycles, and the occupational use of mechanical devices when lifting heavy objects. Secondary prevention is undertaken in cases of latent (hidden) disease. Although the patient may be asymptomatic, the disease process can be detected by medical tests. Nurses may use screening tests to assess for latent disease in vulnerable populations. Examples of screening tests used as secondary prevention strategies include the purified protein derivative (PPD) skin test for tuberculosis, fecal occult blood test for colorectal cancer, and mammograms for breast cancer. Tertiary prevention, also known as the treatment or rehabilitation stage of preventive care, is implemented when a condition or illness is permanent and irreversible. The aim of care is to reduce the number and impact of complications and disabilities resulting from a disease or medical condition. Interventions are intended to reduce suffering caused by poor health and assist the patients in adjusting to incurable conditions. Nursing care is focused on rehabilitation efforts in the tertiary stage of prevention. Holistic care is an approach to applying healing therapies. Nurses participate in holistic care through the use of natural healing remedies and complementary interventions. These include the use of art and guided imagery, therapeutic touch, music therapy, relaxation techniques, and reminiscence.

21. The nurse knows which method to be an appropriate way to tie restraints? a. Knot tied to the bed frame b. Quick-release knot tied to the side rail c. Bow tied to the bed rail d. Quick-release ties attached to the bed frame

ANS: D Restraints should never be tied in a knot because the knot may prohibit a quick exit in the event of an emergency requiring evacuation. Instead, use quick-release ties or mechanisms such as buckles. Restraints are never be tied to side rails because injuries may result when they are raised or lowered. They should be tied to a stable part of the bed such as the frame.

3. The primary healthcare nurse would recommend screening based on known risk factors, because of which action? a. Eliminate the possibility of developing a condition. b. Identify appropriate treatment guidelines. c. Initiate treatment of a condition or disease. d. Make a substantial difference in morbidity and mortality.

ANS: D Screenings are typically indicated and recommended if the effort makes a substantial difference in morbidity and/or mortality of conditions, and they are safe, cost-effective, and accurate. Ideally a screening measure will accurately differentiate individuals who have a condition from those who do not have a condition 100% of the time; however, there may be a false-negative result, or the patient may develop a condition after the screening was conducted. A screening does not specify treatment guidelines; the screen provides results, and the healthcare provider identifies the treatment. The goal of screening is to identify individuals in an early state of a disease so that prompt treatment can be initiated. The screening results are used for this purpose.

22. Self-concept refers to the way in which individuals perceive unchanging aspects of themselves, such as social character, cognitive abilities, physical appearance, and body image. Which additional point does the nurse the nurse recognize as part of the definition of self-concept? a. If negative, self-concept will allow the patient to compensate for weaknesses. b. If positive, self-concept will cause the patient to see challenges as devastating. c. Self-concept is a concept that is derived from the patient internally. d. Self-concept depends on relationships with family and friends.

ANS: D Self-concept refers to the way in which individuals perceive unchanging aspects of themselves, such as social character, cognitive abilities, physical appearance, and body image. It is a mental image of self in relation to others and the surroundings. If the image is positive, the person will develop strengths, compensate for weaknesses, and experience life in a healthy way. If the image is negative (e.g., frail), the person will find life's challenges devastating and sometimes insurmountable. The impact of illness on the self-concept of a patient and the patient's family members depends on how secure the parties' relationships are with one another.

18. The nurse is monitoring for risks for injury identified in the health care environment. Which finding will alert the nurse that these safety risks are occurring? a. Tile floors, cold food, scratchy linen, and noisy alarms b. Dirty floors, hallways blocked, medication room locked, and alarms set c. Carpeted floors, ice machine empty, unlocked supply cabinet, and nurse call system in reach d. Wet floors unmarked, failure to use lift for patient, and alarms not functioning properly

ANS: D Specific risks to a patient's safety within the health care environment include falls, patient-inherent accidents, procedure-related accidents, and equipment-related accidents. Wet floors contribute to falls, pinching finger in door is patient inherent, failure to use the lift is procedure related, and an alarm not functioning properly is equipment related. Tile floors and carpeted or dirty floors do not necessarily contribute to falls. Cold food, ice machine empty, and hallways blocked are not patient-inherent issues in the hospital setting but are more of patient satisfaction, infection control, or fire safety issues. Scratchy linen, unlocked supply cabinet, and medication room locked are not procedure-related accidents. These are patient satisfaction issues and control of supply issues and are examples of actually following a procedure correctly. Noisy alarms nurse call system within reach, and alarms set are not equipment-related accidents but are examples of following a procedure correctly.

4. The nurse is preparing a patient teaching plan and is seeking a way to determine the patient's readiness and motivation to act regarding lifestyle changes to best manage diabetes mellitus. Which model would be useful for this nurse? a. Maslow's hierarchy of needs b. Holistic Health Model c. Health Promotion Model d. Health Belief Model

ANS: D The Health Belief Model was developed by psychologists Hochbaum, Rosenstock, and Kegels. It explores how patients' attitudes and beliefs predict health behavior. Maslow's hierarchy of needs describes the relationships between the basic requirements for survival and the desires that drive personal growth and development. The model is most often presented as a pyramid consisting of five levels. The lowest level is related to physiologic needs, and the uppermost level is associated with self-actualization needs, specifically those related to purpose and identity. Holistic Health Models in nursing care are based on the philosophy that a synergistic relationship exists between the body and the environment. Holistic care is an approach to applying healing therapies. Holistic models focus on the interrelatedness of body and mind. The Health Promotion Model, developed by Pender and colleagues, defines health as a positive, dynamic state of well-being rather than the absence of disease in the physiologic state.

31. The nurse is providing information regarding safety and accidental poisoning to a grandparent who will be taking custody of a 1-year-old grandchild. Which comment by the grandparent will cause the nurse to intervene? a. ―The number for poison control is 800-222-1222.‖ b. ―Never induce vomiting if my grandchild drinks bleach.‖ c. ―I should call 911 if my grandchild loses consciousness.‖ d. ―If my grandchild eats a plant, I should provide syrup of ipecac.‖

ANS: D The administration of ipecac syrup or induction of vomiting is no longer recommended for routine home treatment of poisoning. The nurse must intervene to provide additional teaching. All the rest are correct and do not require follow up. The poison control number is 800-222-1222. After a caustic substance such as bleach has been drunk, do not induce vomiting. This can cause further burning and injury as the substance is eliminated. Loss of consciousness associated with poisoning requires calling 911.

24. A patient may need restraints. Which task can the nurse delegate to a nursing assistive personnel? a. Determining the need for restraints b. Assessing the patient's orientation c. Obtaining an order for a restraint d. Applying the restraint

ANS: D The application and routine checking of a restraint can be delegated to nursing assistive personnel. The skill of assessing a patient's behavior, orientation to the environment, need for restraints, and appropriate use cannot be delegated. A nurse must obtain an order from a health care provider.

23. The patient has the nursing diagnosis of Impaired physical mobility related to pain in the left shoulder. Which priority action will the nurse take? a. Encourage the patient to do self-care. b. Keep the patient as mobile as possible. c. Encourage the patient to perform ROM. d. Assist the patient with comfort measures.

ANS: D The diagnosis related to pain requires the nurse to assist the patient with comfort measures so that the patient is then willing and more able to move. Pain must be controlled so the patient will not be reluctant to initiate movement. The diagnosis related to reluctance to initiate movement requires interventions aimed at keeping the patient as mobile as possible and encouraging the patient to perform self-care and ROM.

14. The nurse has implemented a community-wide immunization program for seasonal influenza. Once the program has ended, what action by the nurse is best? a. Begin planning for next year's program. b. Send mail surveys to participants. c. Determine financial gains or losses. d. Evaluate the program and outcomes.

ANS: D The last step of the nursing process is evaluation. The nurse should evaluate the program to see if interventions had the desired effect. Evaluation could include surveys or looking at financial outcomes, but those are only limited aspects of the process. Planning for next year's event should not occur until after evaluation has been completed.

6. To promote safety, the nurse manager sensitive to point of care (sharp end) and systems level (blunt end) exemplars works closely with staff to address which point of care exemplar? a. Care coordination b. Documentation c. Electronic records d. Fall prevention

ANS: D The most common safety issues at the sharp end include prevention of decubitus ulcers, medication administration, fall prevention, invasive procedures, diagnostic workup, recognition of/action on adverse events, and communication. These are the most common issues the staff nurse providing direct patient care encounters. Each of the other options is classified as systems level exemplars.

4. The nurse is developing an interdisciplinary plan of care using the Roper-Logan-Tierney Model of Nursing for a patient who is currently unconscious. Which interventions would be most critical to developing a plan of care for this patient? a. Eating and drinking, personal cleansing and dressing, working and playing b. Toileting, transferring, dressing, and bathing activities c. Sleeping, expressing sexuality, socializing with peers d. Maintaining a safe environment, breathing, maintaining temperature

ANS: D The most critical aspects of care for an unconscious patient are safe environment, breathing, and temperature. Eating and drinking are contraindicated in unconscious patients. Toileting, transferring, dressing, and bathing activities are BADLs. Sleeping, expressing sexuality, and socializing with peers are a part of the Roper-Logan-Tierney Model of Nursing; however, these are not the most critical for developing the plan of care in an unconscious patient.

2. The nurse is caring for a patient experiencing an allergic reaction to a bee sting who has an order for diphenhydramine (BenaDRYL). The only medication in the patient's medication bin is labeled BenaZEPRIL. The nurse contacts the pharmacy for the correct medication to avoid what type of error? a. Communication b. Diagnostic c. Preventive d. Treatment

ANS: D The nurse avoided a treatment error, giving the wrong medication. Benazepril is an ace inhibitor used to treat blood pressure. The Institute of Medicine (IOM) report referred to Leape's identification of four types of errors. Treatment errors occur in the performance of an operation, procedure, or test; in administering a treatment; in the dose or method of administering a drug; or in avoidable delay in treatment or in responding to an abnormal test. Communication errors refer to those that occur from a failure to communicate. Diagnostic errors are the result of a delay in diagnosis, failure to employ indicated tests, use of outmoded tests, or failure to act on results of monitoring or testing. Preventive errors occur when there is inadequate monitoring or failure to provide prophylactic treatment or follow-up of treatment.

14. Which collaborative team member would be most effective in assisting the nurse to identify medication alternatives that are less likely to cause drowsiness and dizziness to reduce the risk of falls in the elderly patient? a. Nursing case manager b. Charge nurse c. Physical therapist d. Pharmacist

ANS: D The nurse collaborates with the pharmacist and health care provider to identify and implement safe medication alternatives for older adults to minimize side effects such as drowsiness, dizziness, and orthostatic hypotension, which can increase fall risk. Although case managers and charge nurses might have some experience in this area, pharmacists are educated to focus on medication. Physical therapists evaluate the patient's ability to perform and maintain balance during routine activities such as sitting, standing, and walking.

15. The nurse is concerned about helping the patient find resources to obtain assistive equipment to be used in the home. Which team member should the nurse contact first? a. Occupational therapist b. Physical therapist c. Health care provider d. Social worker

ANS: D The nurse should collaborate with the social worker to identify community resources for obtaining assistive equipment. The social worker facilitates contact with insurance companies or other agencies to assist with the financing of recommended therapeutic assistive and specialty devices. Occupational therapists evaluate the patient for safe performance of activities of daily living (ADLs) such as bathing, dressing, and grooming, and they make recommendations to enhance safe performance of these activities, such as the use of specialty equipment (e.g., grippers for pants, oversized shoehorns). Physical therapists evaluate the patient's ability to perform and maintain balance during routine activities such as sitting, standing, and walking. They make recommendations for assistive devices such as canes and walkers to promote safe performance of these activities. Health care providers order the equipment.

27. The patient is confused, is trying to get out of bed, and is pulling at the intravenous infusion tubing. Which nursing diagnosis will the nurse add to the care plan? a. Impaired home maintenance b. Deficient knowledge c. Risk for poisoning d. Risk for injury

ANS: D The patient's behaviors support the nursing diagnosis of Risk for injury. The patient is confused, is pulling at the intravenous line, and is trying to climb out of bed. Injury could result if the patient falls out of bed or begins to bleed from a pulled line. Nothing in the scenario indicates that this patient lacks knowledge or is at risk for poisoning. Nothing in the scenario refers to the patient's home maintenance.

15. The nurse correctly teaches the patient to rise from a chair using crutches when which intervention is used? a. Patient starts from the back of the chair. b. The weak leg is closest to the chair. c. The hand on the strong side holds the hand bar of the crutch. d. The strong leg is closest to the chair.

ANS: D The patient's strongest leg needs to be closest to the chair. The patient's hand on the weak side holds the hand bar of the crutches, and the hand on the patient's strong side holds onto the armrest of the chair. The patient moves to the front edge of the chair.

20. The nurse is caring for an older-adult patient with a diagnosis of urinary tract infection (UTI). Upon assessment the nurse finds the patient confused and agitated. How will the nurse interpret these assessment findings? a. These are normal signs of aging. b. These are early signs of dementia. c. These are purely psychological in origin. d. These are common manifestation with UTIs.

ANS: D The primary symptom of compromised older patients with an acute urinary tract infection or fever is confusion. Acute confusion in older adults is not normal; a thorough nursing assessment is the priority. With the diagnosis of urinary tract infection, these are not early signs of dementia and they are not purely psychological.

3. A nurse administers an incorrect medication to a patient. In reviewing this medication error, the nurse finds out that incorrect medication was placed in the Pyxis system. What type of error has the nurse committed? a. Latent error b. Blunt end c. Did not follow nursing process d. Latent error resulting in active error

ANS: D The situation described is a latent error which resulted in an active error as incorrect medication was placed in the Pyxis system. Latent errors are also referred to as blunt end whereas active errors are applied as occurring at the sharp end. There is no provided information to suggest that the nurse did not follow nursing process.

34. The patient is unable to move self and needs to be pulled up in bed. What will the nurse do to make this procedure safe? a. Place the pillow under the patient's head and shoulders. b. Do by self if the bed is in the flat position. c. Place the side rails in the up position. d. Use a friction-reducing device.

ANS: D This is not a one-person task. Helping a patient move up in bed without help from other co-workers or without the aid of an assistive device (e.g., friction-reducing pad) is not recommended and is not considered safe for the patient or the nurse. Remove the pillow from under head and shoulders and place it at the head of the bed to prevent striking the patient's head against the head of the bed. When pulling a patient up in bed, the bed should be flat to gain gravity assistance, and the side rails should be down.

1. A home health nurse is performing a home assessment for safety. Which comment by the patient will cause the nurse to follow up? a. ―Every December is the time to change batteries on the carbon monoxide detector.‖ b. ―I will schedule an appointment with a chimney inspector next week.‖ c. ―If I feel dizzy when using the heater, I need to have it inspected.‖ d. ―When it is cold outside in the winter, I will use a non-vented heater.‖

ANS: D Using a non-vented heater introduces carbon monoxide into the environment and decreases the available oxygen for human consumption and the nurse should follow up to correct this behavior. Checking the chimney and heater, changing the batteries on the detector, and following up on symptoms such as dizziness, nausea, and fatigue are all statements that are safe and appropriate and need no follow-up.

9. During an assessment, the nurse finds the patient experiences vertigo. Which sensory deficit will the nurse assess further? a. Neurological deficit b. Visual deficit c. Hearing deficit d. Balance deficit

ANS: D Vertigo is a result of vestibular dysfunction and often is precipitated by a change in head position. Neurological deficits include peripheral neuropathy and stroke. Visual deficits include presbyopia, cataracts, glaucoma, and macular degeneration. Hearing deficits include presbycusis and cerumen accumulation.

2. A nurse is describing the transmission of sound to a patient. In which order will the nurse list the pathway of sound, beginning with the first structure? 1. Eardrum 2. Perilymph 3. Oval window 4. Bony ossicles 5. Eighth cranial nerve a. 1, 5, 2, 4, 3 b. 1, 3, 4, 2, 5 c. 1, 2, 4, 5, 3 d. 1, 4, 3, 2, 5

ANS: D Vibration of the eardrum transmits through the bony ossicles. Vibrations at the oval window transmit in perilymph within the inner ear to stimulate hair cells that send impulses along the eighth cranial nerve to the brain.

5. The nurse in a newly opened community health clinic is developing a program for the individuals considered at greatest risk for poor health outcomes. How should the nurse consider this group? a. Global community b. Sedentary society c. Unmotivated population d. Vulnerable population

ANS: D Vulnerable populations refer to groups of individuals who are at greatest risk for poor health outcomes. The entire world is the global community. Sedentary refers to the lifestyles of people worldwide who have epidemic rates of obesity and many other related chronic diseases. Unmotivated population refers to the individuals who have not demonstrated interest in changing.

5. The nurse is completing an admission history on a new home health patient. The patient has been experiencing seizures as the result of a recent brain injury. Which interventions should the nurse utilize for this patient and family? (Select all that apply.) a. Demonstrate how to restrain the patient in the event of a seizure. b. Instruct the family to move the patient to a bed during a seizure. c. Teach the family how to insert a tongue depressor during the seizure. d. Discuss with the family steps to take if the seizure does not discontinue. e. Instruct the family to reorient and reassure the patient after consciousness is regained.

ANS: D, E Prolonged or repeated seizures indicate status epilepticus, a medical emergency that requires intensive monitoring and treatment. Family should know what to do. Family should reorient and reassure the patient after consciousness is regained. Never force apart a patient's clenched teeth. Do not place any objects into patient's mouth such as fingers, medicine, tongue depressor, or airway when teeth are clenched. Do not lift patient from floor to bed while seizure is in progress. Do not restrain patient; hold limbs loosely if they are flailing. Loosen clothing.

10. Which of the following statements adheres to documentation that meets legal and ethical standards? (Select all that apply.) A. Leaving sufficient documentation space for the previous shift to document patient education. B. Documents the patient reports, "I'm so afraid of tomorrow's surgery." C. Documents the patient's response to administered pain medication. D. Marking out error with one line of white out and initials

B. Documents the patient reports, "I'm so afraid of tomorrow's surgery." C. Documents the patient's response to administered pain medication.

11. A nurse is caring for a nonverbal patient and begins to suspect that the patient is in pain. Which nonverbal manifestations would indicate pain? (Select all that apply.) A. Decreased respiratory rate B. Increase blood pressure C. Facial grimacing D. Increased urinary output

B. Increase blood pressure C. Facial grimacing

4. A patient in o health care facility asks the nurse about the facility's computerized system for keeping patient information, especially confidentiality issues. Which is the best response by the nurse? A. "Don't worry, your information is always safe." B. Information In our system requires a password to retrieve." C. Our system was designed with a lot of Input from nursing staff." D. "I can see why you're worried, with all the computer hackers out there these days."

B. Information In our system requires a password to retrieve."

38. An elderly patient presents to the hospital with a history of falls, confusion, and stroke. The nurse determines that the patient is at high risk for falls. Which of the following interventions is most appropriate for the nurse to take? A. Place the patient in restraints. B. Lock beds and wheelchairs when transferring. C. Place a bath mat outside the tub. D. Silence fall alert alarm upon request of family.

B. Lock beds and wheelchairs when transferring.

36. The community health nurse is administering flu vaccines to individuals at a local park. In doing so, what is the nurse's focus? A. Preventing individual Illness. B. Preventing community outbreak of illness. C. Preventing outbreak of illness in the family. D. Administering to the needs of the individual or family

B. Preventing community outbreak of illness.

20. When performing collaborative health care, the nurse must implement which of the following? A. Assume leadership role in directing the health care team. B. Rely on the expertise of other health care team members. C. Be physically present for the implementation of all aspects of the care plan. D. Delegate decision-making authority to each health care provider.

B. Rely on the expertise of other health care team members.

34. A patient is admitted to the local hospital with reports of shortness of breath (50B). The patient is placed on oxygen, as labs and arterial blood gases (ABGs) drawn, and is given an electrocardiogram (ECG). What level of preventive care is this patient receiving A. Primary prevention B. Secondary prevention C. Tertiary prevention D. Health promotion prevention

B. Secondary prevention

42. What is the major difference between a baccalaureate degree nursing (BSN) program and an associate's degree (ASN) program? A. Basic sciences and theoretical courses. B. Social sciences and humanities. C. Theoretical and clinical courses. D. Basic sciences and clinical courses

B. Social sciences and humanities.

29. A homeless adult patient presents to the emergency department. The nurse obtains the following vital signs: temperature 94.8 F, blood pressure 100/56, apical pulse 56, respiratory rate 12. Which of the vital signs should be addressed immediately? A. Respiratory rate B. Temperature C. Apical pulse D. Blood pressure

B. Temperature

24. The nurse is working the night shift on a surgical unit and is making 4 AM rounds. She notices that the patient's temperature is 96.8° F (36° C), whereas at 4 PM the preceding day, it was 98.6° F (37° C). What should the nurse do? A The patient is suffering from hypothermia. B. The patient is demonstrating a normal temperature. C. The patient is hyperthermic relative to their age, D. The patient is demonstrating an Increased metabolism that accompanies aging.

B. The patient is demonstrating a normal temperature.

33. Which health care model utilizes Maslow's hierarchy as its base? A. Health Belief B. Health Promotion C Basic Human Needs D. Holistic Health

C Basic Human Needs

3. A patient comes to the clinic and is found to have a sexually transmitted infection (S TI). The patient states to the nurse, "Promise you won't tell anyone about my condition." According to the Health Insurance Portability and Accountability Act (HIPAA) of 1996, the nurse must do which of the following? A. Honor the patient's wishes B. Not disclose any Information to anyone C. Communicate only necessary information D. Respect the patient's privacy and confidentially

C. Communicate only necessary information

32. A nurse has taught the patient how to use crutches. The patient went up and down the stairs using crutches with no difficulties. Which information will the nurse use for the "I" in PIE documentation? A. Deficient knowledge regarding crutches B. Patient went up and down stairs C. Demonstrated use of crutches D. Used crutches with no difficulties

C. Demonstrated use of crutches

6. A student nurse Is learning how to Implement the nursing process into a plan of care for a patient. What does the student nurse realize about the purpose of the nursing process? A. Deliver care to a patient in an organized and efficient manner. B. Implement a plan that is close to the medical model. C. Identify patent needs and deliver care to meet those needs D. Ensure that standardized care is available to patients,

C. Identify patent needs and deliver care to meet those needs

2. The client presents her hand when the nurse makes this statement: I need to start an IV so you can get your antibiotics. Which type of consent? A. Informed consent B. Expressed consent C. Implied consent D. Compliance consent

C. Implied consent

31. A nurse has provided care to a patient. Which entry should the nurse document in the patient's record? A. "Patient seems to be in pain and states, 'I feel uncomfortable.'" B. Status unchanged, doing well C. Left abdominal incision 1 inch in length without redness, drainage, or edema D. Patient is hard to care for and refuses all treatments and medications. Family present

C. Left abdominal incision 1 inch in length without redness, drainage, or edema

43. The nurse is performing hand hygiene before assisting a physician with insertion of a chest tube. While washing hands, nurse touches the sink. What is the next action the nurse should take? A Inform Che physician and recruit another nurse to assist. B. Rinse and dry hands, and begin assisting the physician. C. Repeat handwashing using antiseptic soap, D. Extend the handwashing procedure to 1 minute.

C. Repeat handwashing using antiseptic soap,

27. Which of the following concepts are important to utilize when evaluating orders for a restraint? (Select all that apply) A. Patient behaviors that necessitate the use of restraint are part of the nursing plan of care. B. A restraint Is removed every 3 hours and the patient's skin is assessed. C. Restraint orders are lime limited and must be renewed by the provider. D. A provider's order is required for restraint and includes a face-to-face evaluation.

C. Restraint orders are lime limited and must be renewed by the provider. D. A provider's order is required for restraint and includes a face-to-face evaluation.

37. During the admission assessment, the nurse assesses the patient for fall risk. Which of the following has the greatest potential to increase the patient's risk for falls? A. The patient is 59 years of age. B. The patient walks 2 miles a day. C. The patient takes Benadryl (diphenhydramine) for allergies. D. The patient recently became widowed.

C. The patient takes Benadryl (diphenhydramine) for allergies.

12. A nurse is assessing a new colostomy stoma. The assessment reveals a beefy, red, and swollen stoma with surrounding area appearing intact. What in the best conclusion made by the nurse? A. The stoma site is developing an infection. B. The patent is developing maceration at the stoma site. C. The stoma site is healing appropriately. D. The stoma ostomy wafer and pouch have been placed too close to the stoma.

C. The stoma site is healing appropriately.

39. The older patient presents to the emergency department after stepping in front of a car at a crosswalk. After the patient has been triaged, the nurse interviews the patient. Which of the following comments would require follow- up by the nurse? A. "I try to exercise, so I walk that block almost every day B. "I walled and stepped out when the traffic sign said go. C. "The car was going too fast, the speed limit is 20." D. "I was so surprised; I didn't see or hear the car coming."

D. "I was so surprised; I didn't see or hear the car coming."

23. A nurse is caring for a patent who smokes and drinks caffeine. Which point is important for the nurse to understand before assessing the patient's blood pressure? A. Neither caffeine nor smoking affects blood pressure. B. The nurse waits until the patient has stopped smoking for at least 30 minutes. C. The nurse should have the patent perform mild exercises. D. Caffeine and smoking can cause false BP elevations.

D. Caffeine and smoking can cause false BP elevations.

25. The patient is found to be unresponsive and not breathing. To determine the presence of central blood circulation and circulation of blood to the brain, the nurse checks the patient's _____ pulse. A. Radial B. Brachial C. Femoral D. Carotid

D. Carotid

5. Nurses must use critical thinking in their day-to-day practice, especially in circumstances surrounding patient care and wise use of resources. In which of the following situations would critical thinking be most beneficial? A. Administering Intravenous (IV) push meds to critically Ill patients B. Teaching new parents about car seat safety C. Assisting an orthopedic patient with the proper use of crutches D. Educating a home health patient about treatment options

D. Educating a home health patient about treatment options

1. The nurse documented the patient's recovery outcomes which were met prior to the goal nurse's documentation follows which of the ANA standards of practice? A Diagnosis B. Planning C. Implementation D. Evaluation

D. Evaluation

21. A student nurse is checking an apical pulse on a patient who has just returned from surgery. Which of the following are the most important elements of this skill? A. Place the diaphragm of the stethoscope at the third intercostal space at the midclavicular line and count for 60 seconds. B. Place the diaphragm of the stethoscope at the fourth intercostal space at the sternal border and count for 60 seconds. C. Place the diaphragm of the stethoscope at fifth intercostal space at the sternal border and count for 60 seconds. D. Place the diaphragm of the stethoscope at the fifth Intercostal space at midclavicular line and count for 60 seconds.

D. Place the diaphragm of the stethoscope at the fifth Intercostal space at midclavicular line and count for 60 seconds.

18. A nurse is caring for a patient with anorexia. Which of the following would be an example of Interpersonal nursing interventions? A Provide basic dental hygiene B. Educate on minimal dietary requirements C. Supervise patient's oral intake D. Provide an opportunity to examine values

D. Provide an opportunity to examine values

44. Which interventions utilized by the nurse will indicate the ability to recognize a localized inflammatory response? A. Vigorous range-of-motion exercises B. Turn, cough, and deep breathe C. Orient to date, time, and place D. Rest, ice, and elevation

D. Rest, ice, and elevation

28. The patient is confused, is trying to get out of bed, and is pulling at the intravenous infusion tubing. Which of the following diagnoses is best supported by the gathered data? A. Risk for illness B. Knowledge deficit C. Impaired home maintenance D. Risk for Injury

D. Risk for Injury

26. A home health nurse is performing a home assessment for safety. Which of the following comments by the patient would indicate a need for further education? A. I will schedule an appointment with a chimney Inspector next week." B. "Daylight savings is the time to change batteries on the carbon monoxide detector." C. "If I feel dizzy when using the heater, I need to have it inspected." D. When it is cold outside in the winter, I can warm my car up in the garage."

D. When it is cold outside in the winter, I can warm my car up in the garage."


Conjuntos de estudio relacionados

communications law midterm (quiz 2)

View Set

CIS - Connect Computer Input Quiz

View Set